Study Exam 2, Ch 12 (Evolve ch. 16, 18, 19)

अब Quizwiz के साथ अपने होमवर्क और परीक्षाओं को एस करें!

A nurse is told in a hand off report that a woman's cervix is not yet ripened. What does the nurse understand about this patient? A. She is ready to deliver. B. Her cervix has not yet softened. C. Vaginal delivery will not be possible. D. This change begins labor in all women.

(Chapter 12, Pg. 413) Toward the end of pregnancy, there is a complete reversal in which the uterus becomes more excitable, and cervical softening (ripening) occurs. The cervical changes result from the breakdown of collagen fibers, which produce a decrease in the binding capacity. This change, coupled with an increase in cervical water content, causes weakening and softening. (B)

The nurse is caring for a multiparous client. In which stage can the nurse expect the fetal head to be engaged in the pelvic inlet? A. About 2 weeks before term B. Before the start of active labor C. When labor stage I begins D. Many weeks before the onset of labor

(Chapter 12, Pg. 420) In a multiparous client, the abdominal musculature is relaxed. The fetal head often remains freely movable above the pelvic brim, and becomes engaged in the pelvic inlet only after labor is established. In a nulliparous client, the uterus sinks downward and forward about 2 weeks before term, when the presenting part of the fetus descends into the true pelvis. The fetal head is engaged in the pelvic inlet before the onset of active labor. The abdominal muscles are firm in a nulliparous pregnancy and direct the presenting part into the pelvis. The first stage of labor lasts from the onset of regular uterine contractions to full dilation of the cervix. (D)

The perinatal nurse describes for the student nurse the lettering used to designate fetal position. The correct use includes A. "P" indicating fetal pelvis location. B. "P" indicating posterior maternal pelvis. C. "M" indicating fetal mandible. D. "A" indicating maternal anus

(Chapter 12, Pg. 421) B. P = posterior. M = mentum. A = Anterior.

A nurse reads in a patient's chart that she has employed a doula. What does the nurse understand about this role? A. Provides physical and emotional support during labor. B. Performs continuous patient assessment during labor. C. Performs private duty nursing care during labor and birth. D. Assists the surgeon during a cesarean delivery.

(Chapter 12, Pg. 426) The douala's role is to provide continuous information and physical and emotional support to the woman and her partner before, during, and immediately after the birth. She does not function in a clinical role but instead specializes in providing comfort measures to decrease the woman's anxiety. (A)

While auscultating for fetal heart tones in a pregnant client, the nurse observes that there is persistent fetal tachycardia. In which situation would this finding be considered normal? A. If the patient's body temperature has increased B. If the tachycardia is caused by late deceleration C. If the tachycardia is related to minimal variability D. If the client's uterine contractions (UCs) are elevated

(Chapter 12, Pg. 443) Accelerations in the fetal heart rate (FHR) are usually episodic and sometimes they may be persistent. When the patient is febrile, fetal tachycardia is not considered as a serious event. The tachycardia would resolve once the patient is afebrile. Elevation in UCs may cause episodic tachycardia, but not persistent tachycardia. Persistent tachycardia when associated with late deceleration or minimal variability is considered a risk to the fetus. (A)

Evidence-based care practices designed to support normal labor and birth recommend which practice during the immediate newborn period? A. The healthy newborn should be taken to the nursery for a complete assessment. B. After drying, the infant should be given to the mother wrapped in a receiving blanket. C. Encourage skin-to-skin contact of mother and baby. D. The father or support person should be encouraged to hold the infant while awaiting delivery of the placenta.

(Chapter 12, pg 463) The unwrapped infant should be placed on the woman's bare chest or abdomen, then covered with a warm blanket. Skin-to-skin contact keeps the newborn warm, prevents neonatal infection, enhances physiologic adjustment to extrauterine life, and fosters early breastfeeding. Although this is the practice in many facilities it is neither evidence-based nor supportive of family-centered care. This is a common practice and more family friendly than separating mother and baby; however, ideally the baby should be placed skin to skin. The father or support person is likely anxious to hold and admire the newborn. This can happen after the infant has been placed skin to skin and breastfeeding has been initiated. (C)

Under which circumstances should a vaginal examination be performed by the nurse? Select all that apply. A. An admission to the hospital at the start of labor. B. When accelerations of the fetal heart rate (FHR) are noted. C. On maternal perception of perineal pressure or the urge to bear down. D. When membranes rupture. E. When bright, red bleeding is observed.

(Chapter 12, pg. 416 & 437) Vaginal examinations should be performed when the woman is admitted to the hospital or birthing center at the start of labor. When the woman perceives perineal pressure or the urge to bear down is an appropriate time to perform a vaginal examination. After rupture of membranes (ROM) a vaginal examination should be performed. The nurse must be aware that there is an increased risk of prolapsed cord immediately after ROM. An accelerated FHR is a positive sign; variable decelerations, however, merit a vaginal examination. Examinations are never done by the nurse if vaginal bleeding is present because the bleeding could be a sign of placenta previa and a vaginal examination could result in further separation of the low-lying placenta. (A, C, D)

Which position would the nurse suggest for second-stage labor if the pelvic outlet needs to be increased? A. Semirecumbent B. Sitting C. Squatting D. Side-lying

(Chapter 12, pg. 435) Kneeling or squatting moves the uterus forward and aligns the fetus with the pelvic inlet; this can facilitate the second stage of labor by increasing the pelvic outlet. Sitting may assist with fetal descent, but like a semirecumbent or side-lying position, it does not increase the size of the pelvic outlet. (C)

The nurse instructor is teaching a group of students about the structure of the fetal head during labor and birth. Which statement by the student indicates effective learning? A. "The fetal skull bones are firmly united during labor." B. "The fetal skull bones are united by membranous sutures." C. "The two important fontanels are the parietal and temporal." D. "The sutures and fontanels restrict brain growth after delivery."

(Chapter 12, pg. 417) Membranous sutures, including sagittal, lambdoidal, coronal, and frontal, unite the fetal skull bones. The fetal skull bones are flexible at birth and a slight overlapping of the bones may occur during labor. The two important fontanels are the anterior and posterior. The sutures and fontanels do not close at birth but instead allow flexibility to accommodate the brain, which continues to grow after birth. (B)

Which pregnant client is likely to have a cesarean delivery? A. A client with the fetus in a transverse lie B. A client with the fetus in a cephalic presentation C. A client with the fetal biparietal diameter of 9.25 cm at term D. A client in whom the presenting part is 4 cm below the spines

(Chapter 12, pg. 420) A transverse lie indicates that the long axis of the fetus is at a right angle, diagonal to the long axis of the mother. As a result, a vaginal birth is not possible and the client will need a cesarean delivery. A cephalic presentation indicates that the fetal head will lead through the birth canal during labor. This presentation facilitates vaginal delivery. A fetal biparietal diameter of 9.25 cm indicates normal head growth, which can be easily delivered vaginally. If the presenting part is 4 cm below the spines, it indicates that birth is imminent. The part is not an indicator of the type of birth. (A)

The nurse assisting a laboring client is aware that the birth of the fetus is imminent. What is the station of the presenting part? A. -1 B. +1 C. +3 D. +5

(Chapter 12, pg. 420) Station is the relationship of the presenting fetal part to an imaginary line drawn between the maternal ischial spines. The placement of the presenting part is measured in centimeters above or below the ischial spines. Birth is imminent when the presenting part is at +4 to +5 cm. When the lowermost portion of the presenting part is 1 cm above the spine, it is noted as minus (-)1. When the presenting part is 1 cm below the spine, the station is said to be plus (+)1. At +3, the presenting part is still descending the birth canal. (D)

During a sterile vaginal exam, the nurse finds that the fetal position is ROA. What is the presenting part of the fetus? A. Occiput B. Sacrum C. Scapula D. Mentum

(Chapter 12, pg. 421) The presenting part of the fetus is the part that appears first during the labor. The fetal position refers to the presenting part in relation to the mother's pelvis. The position is denoted by a three-part abbreviation. In this case, the letters ROA stand for right, occiput, and anterior. It means that the occiput is the presenting part and is located in the right anterior quandrant of the maternal pelvis. Sacrum will be denoted by the letter S. Scapula (shoulder) is denoted by Sc. Mentum (chin) is denoted by the letter M. (A)

The nurse is assessing a pregnant client who is due in 2 weeks. Which signs and symptoms preceding labor may the nurse expect to see in the client? Select all that apply. A. Loss of weight B. Pain in the groin C. Persistent low backache D. Loss of energy E. Blood-tinged cervical mucus

(Chapter 12, pg. 423) The pregnant client may have a weight loss of 0.5 to 1.5 kg in the days preceding labor, due to water loss from electrolyte shifts, caused by changes in estrogen and progesterone levels. Pain in the groin and persistent low backache may occur due to the relaxation of the pelvic joints. The extreme congestion of the vaginal mucous membranes may cause blood-tinged cervical mucus. A surge of energy is a common phenomenon in a pregnant client preceding labor. (A, B, C, D)

Which characteristic is associated with false labor contractions? A. Painless B. Decrease in intensity with ambulation C. Regular pattern of frequency established D. Progressive in terms of intensity and duration

(Chapter 12, pg. 425) Although false labor contractions decrease with activity, true labor contractions are enhanced or stimulated with activity such as ambulation. True labor contractions are painful. A regular pattern of frequency is a sign of true labor. A progression of intensity and duration indicates true labor. (B)

Which type of vaginal discharge is more common during the transition phase of the first stage of labor? A. Bloody mucus B. Pale pink mucus C. Brownish discharge D. Pink-to-bloody mucus

(Chapter 12, pg. 432) During the transition phase, the cervix is dilated to 8 to 10 cm, which results in the appearance of bloody mucus in the vaginal discharge. Pale pink mucus is seen in the latent phase, in which the cervix dilates to not more than 3 cm. Brownish discharge may also be observed during the latent phase of the first stage of labor. Pink-to-bloody mucus is observed in the active phase of cervical dilation. (A)

Nurses can help their clients by keeping them informed about the distinctive stages of labor. What description of the phases of the first stage of labor is accurate? A. Latent: mild, regular contractions; no dilation; bloody show; duration of 2 to 4 hours. B. Active: moderate, regular contractions; 4 to 7 cm dilation; duration of 3 to 6 hours. C. Lull: no contractions; dilation stable; duration of 20 to 60 minutes D. Transition: very strong but irregular contractions; 8 to 10 cm dilation; duration of 1 to 2 hours

(Chapter 12, pg. 432, Table 12-3) The latent phase is characterized by mild to moderate, irregular contractions; dilation up to 3 cm; brownish to pale pink mucus; and a duration of 6 to 8 hours. No official lull phase exists in the first stage. The transition phase is characterized by strong to very strong, regular contractions; 8 to 10 cm dilation; and a duration of 20 to 40 minutes. (B)

Which nursing intervention is important when providing care for a pregnant client in the first stage of the labor? A. Determining the station B. Obtaining an order for epidural anesthesia C. Encouraging the client to hold breath and tighten abdominal muscles D. Encouraging the client to change position frequently

(Chapter 12, pg. 434) In the first stage of labor, the pregnant client may experience fatigue and discomfort. Therefore, the nurse instructs the client to change position frequently to relieve fatigue, increase comfort, and improve circulation. The best time to determine the station is when the labor begins, because it helps to accurately determine the rate of fetal descent. Epidural anesthesia is administered only if the labor is prolonged and the client is unable to tolerate pain. The client may tighten the abdominal muscles or hold her breath in the second stage of labor. However, the nurse should advise the client against it to prevent fetal hypoxia. (D)

The nurse caring for a laboring woman is aware that maternal cardiac output can be increased by: A. Change in position. B. Oxytocin administration. C. Regional anesthesia. D. Intravenous analgesic.

(Chapter 12, pg. 435) Maternal supine hypotension syndrome is caused by the weight and pressure of the gravid uterus on the ascending vena cava when the woman is in a supine position. This pressure reduces venous return to the woman's heart, as well as cardiac output, and subsequently lowers her blood pressure. The nurse can encourage the woman to change positions and avoid the supine position. Oxytocin administration, regional anesthesia, and intravenous analgesic may all reduce maternal cardiac output. (A)

A woman who is 39 weeks pregnant expresses fear about her impending labor and how she will manage. What is the nurse's best response? A. "Don't worry about it. You'll do fine." B. "It's normal to be anxious about labor. Let's discuss what makes you afraid." C. "Labor is scary to think about, but the actual experience isn't." D. "You may have an epidural. You won't feel anything."

(Chapter 12, pg. 436) Discussing the woman's fears allows her to share her concerns with the nurse and is a therapeutic communication tool. Telling the woman not to worry negates her fears and is not therapeutic. Telling the woman that labor is not scary negates her fears and offers a false sense of security. A number of criteria must be met for use of an epidural. Furthermore, many women still experience the feeling of pressure with an epidural. (B)

A woman who is 39 weeks pregnant expresses fear about her impending labor and how she will manage. The nurse's best response is: A. "Don't worry about it. You'll do fine." B. "It's normal to be anxious about labor. Let's discuss what makes you afraid." C. "Labor is scary to think about, but the actual experience isn't." D. "You may have an epidural. You won't feel anything."

(Chapter 12, pg. 436) This statement allows the woman to share her concerns with the nurse and is a therapeutic communication tool. The statement in A negates the woman's fears and is not therapeutic. The statement in C also negates the woman's fears and offers a false sense of security. The statement in D is not true. A number of criteria must be met for use of an epidural. Furthermore, many women still experience the feeling of pressure with an epidural. (B)

The nurse is auscultating and documenting the fetal heart rate (FHR) of a client intermittently with a handheld Doppler device. Which terms are appropriate for the nurse to use while documenting the FHR? Select all that apply. A. Bradycardia B. Regular rhythm C. Moderate variability D. Variable deceleration E. Absence of acceleration

(Chapter 12, pg. 440, Legal Alert) Bradycardia is a baseline FHR less than 110 beats/minute for 10 minutes or longer. The nurse can use the term "bradycardia," which is numerically defined, when documenting the FHR. When the FHR is auscultated by handheld doptone, it should be described as a baseline number or range and as having a regular or irregular rhythm. The term "absence of acceleration" can also be used by the nurse documenting the FHR. The presence or absence of accelerations or decelerations, both during and after contractions, should also be noted. Descriptive terms associated with EFM such as "moderate variability" and "variable deceleration" cannot be used. They are visual descriptions of the patterns produced on the monitor tracing. The term "variable deceleration" cannot be used while documenting the FHR; it is a visually abrupt decrease in the FHR below the baseline. (A, B, E)

The nurse is caring for a client with electronic fetal monitoring using a spiral electrode. How is the use of a spiral electrode different from the use of an ultrasound transducer? A. It is used only during the antepartum period. B. It is used when the cervix has not yet dilated. C. It is applied firmly to the maternal abdomen. D. It is used after the membranes have ruptured.

(Chapter 12, pg. 442) A spiral electrode can be used only after the membranes have ruptured. The electrode is attached securely to the presenting fetal body part to obtain a good signal. It can be used only during the intrapartum period and only if the cervix is sufficiently dilated and the membranes are ruptured. A tocotransducer is applied firmly to the maternal abdomen to monitor the frequency and duration of contractions. A spiral electrode penetrates into the presenting part by 1.5 mm. (D)

Which device should the nurse use for monitoring the intensity of uterine contractions (UCs) in a pregnant client? A. Tocotransducer B. Spiral electrode C. Ultrasound transducer D. Intrauterine pressure catheter (IUPC)

(Chapter 12, pg. 442) An IUPC measures the frequency, duration, and intensity of contractions during the intrapartum period. The device records the pressure at the catheter tip, and the values are expressed in terms of mm Hg. However, for accurate readings the membranes should be ruptured and the cervix should be dilated. A tocotransducer monitors the frequency and duration of contractions for both antepartum and intrapartum care. This device is placed on the abdomen of the client. Spiral electrodes and ultrasound transducers are used for assessing the fetal heart rate, not the intensity of contractions. A spiral electrode is an invasive mode, whereas an ultrasound transducer is a noninvasive mode. (D)

The nurse is assessing the fetal heart rate in a pregnant client with diabetes during the first stage of labor. At what time intervals should the nurse perform FHR tracing? A. 5 minutes B. 60 minutes C. 15 minutes D. 30 minutes

(Chapter 12, pg. 442) Diabetes is one of the risk factors in pregnancy. If any risk factors are present, the FHR tracing should be evaluated more frequently (every 15 minutes) in the first stage of labor and every 5 minutes in the second stage of labor. FHR should not be evaluated every 1 hour either in low-risk or high-risk clients. In low-risk clients the FHR tracing should be evaluated for every 30 minutes during the first stage of labor. (C)

A laboring woman's temperature is elevated as a result of an upper respiratory infection. The FHR pattern that reflects maternal fever is: A. Diminished variability. B. Variable decelerations. C. Tachycardia. D. Early decelerations.

(Chapter 12, pg. 443) The FHR increases as the maternal core body temperature elevates; therefore tachycardia is the pattern exhibited; it is often a clue of intrauterine infection because maternal fever is often the first sign; diminished variability reflects hypoxia, variable decelerations are characteristic of cord compression, and early decelerations are characteristic of head compression by the cervix. (C)

*Challenge* Fetal bradycardia is most common during: A. Maternal hyperthyroidism. B. Fetal anemia. C. Viral infection. D. Tocolytic treatment using ritodrine.

(Chapter 12, pg. 444) Fetal bradycardia can be considered a later sign of fetal hypoxia and is known to occur before fetal death. Bradycardia can result from placental transfer of drugs, viral infections such as cytomegalovirus (CMV), maternal hypothermia, and maternal hypotension. Maternal hyperthyroidism, fetal anemia, and tocolytic treatment using ritodrine will most likely result in fetal tachycardia. (C)

The nurse is monitoring the fetal heart rate (FHR) of a client. When would the nurse observe early decelerations? A. During uterine contractions B. When external sound is applied C. When the abdomen is palpated D. During regular fetal movement

(Chapter 12, pg. 446) Compression of the fetal head during uterine contraction can cause early decelerations. Fetal heart rate accelerations occur in response to applying external sounds. Palpation of the abdomen also causes FHR accelerations, but not decelerations. Spontaneous and regular fetal movement indicates fetal well-being and results in FHR accelerations. (A)

The nurse providing care for the laboring woman understands that accelerations with fetal movement: A. Are reassuring. B. Are caused by umbilical cord compression. C. Warrant close observation. D. Are caused by uteroplacental insufficiency.

(Chapter 12, pg. 446) Episodic accelerations in the fetal heart rate (FHR) occur during fetal movement and are indications of fetal well-being; they do not warrant close observation. Umbilical cord compression results in variable decelerations in the FHR. Uteroplacental insufficiency would result in late decelerations in the FHR. (A)

The nurse administers an amnioinfusion to a pregnant client according to the primary health care provider's (PHP) instructions. What is the reason behind the PHP's instructions? A. Late decelerations B. Early decelerations C. Variable decelerations D. Prolonged decelerations

(Chapter 12, pg. 447, Table 12-6) Variable decelerations in the FHR are observed when the umbilical cord is compressed. An amnioinfusion refers to the infusion of isotonic fluid into the uterine cavity when the amniotic fluid levels are decreased. This intervention is usually done for the prevention of umbilical cord compression. Late decelerations are observed when infections or elevated uterine contractions (UCs) are seen in a patient. This condition will be reversed by maintaining IV solution, but aminoinfusion is not administered. Early deceleration in the FHR is a normal sign that does not require any intervention. Prolonged deceleration of the FHR occurs when there is marked reduction of fetal oxygen supply. (C)

The nurse notes variable fetal heart rate (FHR) decelerations while monitoring the fetal heart rate of a patient. What causes variable decelerations? A. Uterine tachysystole B. Maternal hypertension C. Umbilical cord compression D. Epidural or spinal anesthesia

(Chapter 12, pg. 448) Variable FHR decelerations are usually transient and correctable. They can occur at any time during the uterine contraction phase and are caused by umbilical cord compression. Uterine tachysystole is a condition that causes frequent uterine contractions, often more than five contractions in 10 minutes. This causes disruption of oxygen transfer from the environment to the fetus, leading to late decelerations. Maternal hypertension leads to late FHR decelerations due to reduced oxygen transfer to the fetus. Epidural or spinal anesthesia reduces blood flow through maternal vessels, causing late decelerations. (C)

The nurse observes late decelerations in the fetal heart rate (FHR) while caring for a client in labor. Which nursing intervention does the nurse perform for this client? A. Arrange for internal fetal heart rate monitoring. B. Increase the dosage of exogenous oxytocin. C. Provide external sound stimulation. D. Assist the client to knee chest position.

(Chapter 12, pg. 448, Table 12-6) If the nurse notices late FHR decelerations, then the nurse must arrange for internal monitoring to obtain a more accurate fetal and uterine assessment. The nurse may need to help prepare for cesarean birth if the pattern cannot be corrected. The nurse must discontinue any infusion of oxytocin, because it may cause uterine hypertonus, leading to reduction in blood flow to the intervillous space in the placenta, causing fetal hypoxia. The nurse only provides external sound stimulation in order to elicit FHR accelerations during an NST test. The client must be assisted to a lateral position, not a knee-to-chest position. The knee-to-chest position is used if the nurse suspects umbilical cord compression. (A)

Which description of the phases of the second stage of labor is accurate? A. Latent phase: feels sleepy, fetal station is 2+ to 4+, duration is 30 to 45 minutes B. Active phase: overwhelmingly strong contractions, Ferguson reflux activated, duration is 5 to 15 minutes C. Descent phase: significant increase in contractions, Ferguson reflux activated, average duration varies D. Transitional phase: woman "laboring down," fetal station is 0, duration is 15 minutes

(Chapter 12, pg. 454) The descent phase begins with a significant increase in contractions, the Ferguson reflex is activated, and the duration varies, depending on a number of factors. The latent phase is the lull, or "laboring down" period, at the beginning of the second stage. It lasts 10 to 30 minutes on average. The second stage of labor has no active phase. The transition phase is the final phase in the first stage of labor; contractions are strong and painful. (C)

Which description of the phases of the second stage of labor is accurate? A. Latent phase: feels sleepy, fetal station is 2+ to 4+, duration is 30 to 45 minutes. B. Active phase: overwhelmingly strong contractions, Ferguson reflux activated, duration is 5 to 15 minutes. C. Descent phase: significant increase in contractions, Ferguson reflux activated, average duration varies. D. Transitional phase: woman "laboring down," fetal station is 0, duration is 15 minutes

(Chapter 12, pg. 454) The descent phase begins with a significant increase in contractions, the Ferguson reflex is activated, and the duration varies, depending on a number of factors. The latent phase is the lull, or "laboring down," period at the beginning of the second stage. It lasts 10 to 30 minutes on average. The second stage of labor has no active phase. The transition phase is the final phase in the second stage of labor; contractions are strong and painful. (C)

The nurse assisting a laboring client recognizes the Ferguson reflex in the patient. What is the Ferguson reflex? A.Release of endogenous oxytocin B. Involuntary uterine contractions C. Maternal urge to bear down D.Mechanical stretching of the cervix

(Chapter 12, pg. 454) The maternal urge to bear down is known as the Ferguson reflex. The Ferguson reflex is occurs when stretch receptors in the posterior vagina cause release of endogenous oxytocin. The involuntary uterine contractions or primary powers originate at certain pacemaker points in the thickened muscle layers of the upper uterine segment. Intrauterine pressure caused by contractions exerts pressure on the descending fetus and the cervix. When the presenting part of the fetus reaches the perineal floor, mechanical stretching of the cervix occurs. (C)

When managing the care of a woman in the second stage of labor, the nurse uses various measures to enhance the progress of fetal descent. What measures are included? A. Encouraging the woman to try various upright positions, including squatting and standing. B. Telling the woman to start pushing as soon as her cervix is fully dilated. C. Continuing an epidural anesthetic so that pain is reduced and the woman can relax. D. Coaching the woman to use sustained, 10- to 15-second, closed-glottis bearing-down efforts with each contraction.

(Chapter 12, pg. 454) Upright positions and squatting may enhance the progress of fetal descent. Many factors dictate when a woman will begin pushing. Complete cervical dilation is necessary, but it is only one factor. If the fetal head is still in a higher pelvic station, the physician or midwife may allow the woman to "labor down" (allowing more time for fetal descent, thereby reducing the amount of pushing needed) if she is able. The epidural may mask the sensations and muscle control needed for the woman to push effectively. Closed-glottic breathing may trigger the Valsalva maneuver, which increases intrathoracic and cardiovascular pressure, reducing cardiac output and inhibiting perfusion of the uterus and placenta. In addition, holding the breath for longer than 5 to 7 seconds diminishes the perfusion of oxygen across the placenta, resulting in fetal hypoxia. (A)

The nurse is assisting the primary health care provider during the labor process. Which lacerations does the nurse expect in a client who has suffered perineal lacerations? A. Vaginal lacerations B. Cervical lacerations C. Urethral lacerations D. Vaginal vault lacerations

(Chapter 12, pg. 455) A laceration is an irregular tearing of tissues during childbirth. Because the perineum lies below the vagina, lacerations to the perineum may also affect the vagina. Therefore, vaginal lacerations are often associated with perineal lacerations. Cervical lacerations occur at the lateral angles of the external os when the cervix retracts over the advancing fetal head. Urethral lacerations are uncommon or rare during a normal labor process, because they are situated anteriorly. Vaginal vault lacerations may result from rapid fetal descent, precipitous birth, or the use of forceps to rotate the fetal head. (A)

A client sustained perineal lacerations involving the anterior rectal wall during childbirth. What is the severity of the client's perineal laceration? A. First degree B. Second degree C. Third degree D. Fourth degree

(Chapter 12, pg. 455) Perineal lacerations usually occur as the fetal head is being born. The extent of the laceration is defined in terms of its depth. Lacerations that also involve the anterior rectal wall are classified as fourth degree. Lacerations that extend through the skin and the structures superficial to muscles are classified as first degree. Lacerations that extend through the muscles of the perineal body are classified as second degree. Lacerations that continue through the anal sphincter muscle are classified as third degree. (D)

In which stage of labor does the nurse expect the placenta to be expelled? A. First B. Second C. Third D. Fourth

(Chapter 12, pg. 460) The placenta is expelled in the third stage of labor. The placenta normally separates with the third or fourth strong uterine contraction after the infant has been born. The first stage of labor lasts from the time dilation begins to the time when the cervix is fully dilated. The second stage of labor lasts from the time of full cervical dilation to the birth of the infant. The fourth stage of labor lasts for the first 2 hours after birth. (C)

On assessment, the nurse notices that the fetal heart rate is 100 beats/minute. What could be a possible cause for this condition? A. Maternal hypoglycemia. B. Chorioamnionitis. C. Low fetal oxygen supply. D. Decreased fetal hemoglobin levels.

A FHR of less than 110 beats/minute is referred to as bradycardia. Maternal hypoglycemia is a common cause of bradycardia in the fetus. Chorioamnionitis is an infection that also causes tachycardia in the fetus. Hypoxemia and anemia (decreased hemoglobin) of the fetus cause tachycardia, in which the heart rate is more than 160 beats/minute. (A)

Which of the following findings would be a cause for concern for a nurse who is monitoring an obstetric patient who is in early labor? (Select all that apply.) A. Biparietal diameter of less than 9.25 cm B. Vertex presenting part C. Transverse lie D. General flexion attitude E. Android pelvis

A biparietal diameter at term is typically noted as 9.25 cm, and the finding of a smaller measurement would cause a concern related to the mode of delivery. A transverse lie would also cause a concern relative to the mode of delivery because a cesarean section would be indicated. An android pelvis would cause a concern related to the mode of delivery. A vertex presenting part and a general flexion attitude are normal findings and would not cause concern. (A, C, E)

Which of the following findings would be a cause for concern for a nurse who is monitoring an obstetrical client who is in early labor? Select all that apply. A. Biparietal diameter of less than 9.25 cm B. Vertex presenting part C. Transverse lie D. General flexion attitude E. Android pelvis

A biparietal diameter at term is typically noted at 9.25 cm and a finding that this measurement is less than that would cause concern related to the mode of delivery. A transverse lie would cause a concern relative to the mode of delivery, because a C section would be indicated. An android pelvis would cause a concern related to the mode of delivery. A vertex presenting part is a normal finding and would not cause concern. A general flexion attitude is a normal finding and would not cause concern. (A, C, E)

*Challenge* Not in book While caring for a client in labor, the nurse cleans the client's teeth with an ice-cold wet washcloth. What is the rationale behind the intervention? Select all that apply. A. To aid in relaxation B. To refresh the mouth C. To reduce the feeling of thirst D. To reduce the risk of infection E. To counteract dry mouth

A client in labor may not be able to take care of her oral hygiene. Therefore, the nurse should use an ice-cold wet washcloth to clean the client's teeth to maintain oral hygiene and refresh the mouth. The pregnant client may feel thirsty due to active labor. Cleaning the client's teeth with an ice-cold wet washcloth may moisten the oral cavity and reduce the feeling of thirst. This intervention also prevents the oral cavity form becoming dry. Cleaning the teeth does not provide relaxation during labor. The nurse may teach the client deep breathing exercises to promote relaxation during labor. Cleaning the teeth does not reduce infections during labor; however, regular handwashing may decrease the risk of infections. (B, C, E)

What behavior does the nurse expect in a client who is in the transition phase during the first stage of labor? A. The client remains calm and silent. B. The client doubts her ability to control pain. C. The client vomits. D. The client's attention is directed inward.

A client in the transition phase of the first stage of labor has strong uterine contractions, resulting in severe pain. The client may hyperventilate, resulting in nausea and vomiting. The client may remain calm and silent in the latent phase of uterine contractions, because the urge to bear down is not too strong in this phase. During the active stage of labor, the client may become doubtful of her ability to control pain. The client's attention is directed inward in the active phase of the first stage of labor. (C)

After observing the fetal heart activity in the electronic fetal monitor, the nurse suspects that the client's umbilical cord is compressed. What did the nurse observe on the monitor? A. Variable decelerations B. Increase in the fetal heart rate C. Decrease in the fetal heart rate D. Early decelerations

A compressed umbilical cord is commonly observed at the time of labor, which can be determined by variable decelerations in the FHR. Variable decelerations are seen as W- or U-shaped waves on the monitor. The heart rate is not affected due to umbilical cord compression. Early decelerations in the FHR are observed when head of the fetus is compressed. (A)

Fetal monitoring of a pregnant client revealed a regular smooth, undulating wavelike pattern of the fetal heart rate. What should the nurse infer about the fetus from these results? A. Anemia. B. Ischemia. C. Hypertension. D. Hypotension.

A regular smooth, undulating wavelike pattern in the FHR is referred to as a sinusoidal pattern. This uncommon pattern mostly occurs with severe fetal anemia. Ischemia refers to impaired circulation. FHR pattern cannot indicate this condition. Blood pressure level below 120/80 mmHg indicates hypotension. Blood pressure level above 120/80 mm Hg indicates hypertension. The FHR pattern cannot indicate conditions such as hypotension or hypertension. (A)

While monitoring the fetal heart rate (FHR) of a client, the nurse notes a regular, smooth, undulating wavelike pattern. Which condition does the nurse suspect when observing this pattern? A. Prematurity B. Tachycardia C. Fetal anemia D. Neurologic injury

A sinusoidal pattern is a regular, smooth, undulating wavelike pattern. This pattern classically occurs with severe fetal anemia. Minimal variability in the FHR may indicate prematurity or tachycardia. Absent variability may indicate preexisting neurologic injury or congenital anomalies. (C)

Which sign does not precede the onset of labor? A. A return of urinary frequency as a result of increased bladder pressure B. Persistent low backache from relaxed pelvic joints C. Stronger and more frequent uterine (Braxton Hicks) contractions D. A decline in energy, as the body stores up for labor.

A surge of energy is a phenomenon that is common in the days preceding labor. After lightening, a return of the frequent need to urinate occurs as the fetal position causes increased pressure on the bladder. In the run-up to labor, women often experience persistent low backache and sacroiliac distress as a result of relaxation of the pelvic joints. Prior to the onset of labor, it is common for Braxton Hicks contractions to increase in both frequency and strength; bloody show may be passed. (D)

The most common cause of decreased variability in the FHR that lasts 30 minutes or less is: A. Altered cerebral blood flow. B. Fetal hypoxemia. C. Umbilical cord compression. D. Fetal sleep cycles.

A temporary decrease in variability can occur when the fetus is in a sleep state. These sleep states do not usually last longer than 30 minutes. Altered fetal cerebral blood flow results in early decelerations in the FHR, and umbilical cord compression in variable decelerations. Fetal hypoxemia is evidenced by tachycardia initially and then bradycardia. A persistent decrease or loss of FHR variability may be seen. (D)

The most common cause of decreased variability in the FHR that lasts 30 minutes or less is what? A. Altered cerebral blood flow B. Fetal hypoxemia C. Umbilical cord compression D. Fetal sleep cycles

A temporary decrease in variability can occur when the fetus is in a sleep state. These sleep states do not usually last longer than 30 minutes. Altered fetal cerebral blood flow results in early decelerations in the FHR. Fetal hypoxemia is evidenced by tachycardia initially and then bradycardia. A persistent decrease or loss of FHR variability may be seen. Umbilical cord compression results in variable decelerations in the FHR. (D)

The nurse is caring for a client who is in labor. Which interventions would promote safety when assisting the patient with showering? Select all that apply. A. Assess for progress in labor. B. Pour warm water over the client's vulva. C. Place the client's hand in warm water. D. Supervise showers during true labor. E. Help the client in oral hygiene

A warm shower may help relieve discomfort in pregnant client who are in labor. Before the client showers, the nurse should first assess her progress in labor to determine if the client is close to delivery. The nurse should supervise the client during the shower to prevent a fall. Pouring warm water over the client's vulva and placing her hand in warm water during a shower may not promote safety. These interventions may help the client to urinate. Assisting the client in oral hygiene may promote oral health, but may not be helpful in promoting safety. (A, D)

A primigravida calls the hospital and tells a nurse on the labor unit that she knows she is in labor. the nurse's initial response is: A. "Tell me why you know that you are in labor." B. "How far do you live from the hospital?" C. "When is your expected date of birth?" D. "Have your membranes ruptured?"

A. "Tell me why you know that you are in labor." Although choices B, C, and D are all important questions, the first question should gather information regarding whether or not the woman is in labor.

A vaginal examination during labor reveals the following information: LOA, -1, 75%, 3 cm. An accurate interpretation of these data include: (Circle all that apply) A. Attitude: flexed. B. Station: 3 cm below the ischial spines. C. Presentation: cephalic. D. Lie: longitudinal E. Effacement: 75% complete F. Dilation: 9 cm or more to reach full dilation

A. Attitude: flexed. C. Presentation: cephalic. D. Lie: longitudinal E. Effacement: 75% complete (Chapter 12, pg. 415, 418, & 421) The presenting part is 1 cm above the ischial spines as indicated by the "-1"; attitude is flexion of head and neck, as indicated by the occiput ("O") as the presenting part; the lie is longitudinal, as indicated by the cephalic presentation; effacement is 75% complete and is 3 cm dilated, requiring 7 cm more for full dilation to 10 cm.

A nurse assesses the level of a fetal presenting part in a laboring woman at station 0. What does this finding indicate? A. Engagement has occurred. B. The presenting part is above the maternal ischial spines. C. Labor is not progressing. D. The presenting part is at the pelvic outlet.

A. Engagement has occurred. (Chapter 12, Pg. 420) Engagement has occurred when the presenting part is at station zero. When the presenting part lies above the maternal ischial spines, it is at a minus station indicating that the presenting part is at the pelvic inlet. A presenting part below the level of the ischial spines is considered to be a positive station and indicates that the presenting part is at the pelvic outlet.

*Challenge* (Not in our Text) Changes occur as a woman progresses through labor. What maternal adaptations are expected during labor? (Circle all that apply) A. Increase in both systolic and diastolic blood pressure during uterine contractions. B. Decrease in white blood cell count. C. Slight increase in baseline pulse and respiratory rates. D. Decrease in gastric motility leading to nausea and vomiting, especially during the transition phase. E. Proteinuria 2+. F. Hyperglycemia.

A. Increase in both systolic and diastolic blood pressure during uterine contractions. C. Slight increase in baseline pulse and respiratory rates. D. Decrease in gastric motility leading to nausea and vomiting, especially during the transition phase. (Box 16-2 of Lowdermilk & Perry) White blood cell count increases; a decrease in blood glucose and proteinuria of 1+ can be expected.

When admitting a primigravida to the labor unit, the nurse observes for signs that indicate that the woman is in true labor and should be admitted. The nurse recognizes which of the following signs as indicative of true labor? (circle all that apply) A. Woman reports that her contractions seem stronger since she walked from the car to her room on the labor unit. B. Cervix feels soft and is 50% effaced C. Woman perceives pain to be in her back or abdomen above the level of the navel. D. Fetus is engaged in the pelvis at zero station. E. Cervix is in the posterior position. F. Woman continues to feel her contractions intensify following a back rub and the use of effleurage.

A. Woman reports that her contractions seem stronger since she walked from the car to her room on the labor unit. B. Cervix feels soft and is 50% effaced D. Fetus is engaged in the pelvis at zero station. E. Cervix is in the posterior position. Pain of true labor is usually felt in the lower back radiating to the lower portion of the abdomen.

When describing the "powers" of labor to a new nurse, the perinatal nurse discusses the uterine contractions and the A. Woman's pushing efforts. B. Unique musculature of the uterus. C. Position of the fetus D. Hormonal influences regulating labor

A. Woman's pushing efforts. (Chapter 12, Pg. 413) The powers are the physiological forces and birth that include the uterine contractions and the maternal pushing efforts.

During a prenatal evaluation, the nurse notes that the client has a flat pelvis. What term does the nurse use to refer to this type of pelvis? A. Gynecoid B. Android C. Anthropoid D. Platypelloid

About 3% of women may have flat pelvis, which is referred to as platypelloid pelvis. It is flattened anteroposteriorly and wide transversely. About 50% of women have gynecoid pelvis or the classic female type of pelvis. It is slightly ovoid or transversely rounded. Android pelvis resembles the male pelvis and may be found in 23% of women. It is heart shaped or angulated. Anthropoid pelvis resembles the pelvis of anthropoid apes and may be found in 24% of women. It is oval and wider anteroposteriorly. (D)

The nurse is monitoring the fetal heart rate (FHR) of a client in term labor. The FHR varies between 120 and 130 beats/minute over a 10-minute period. How does the nurse record the baseline? Record your answer using a whole number. ________ beats/minute

After 10 minutes of tracing is observed, the approximate mean rate is rounded to the closest 5 beats/minute interval, which is 125 beats/minute. (125)

Nurses can advise their clients that which of these signs precede labor? Select all that apply. A. A return of urinary frequency as a result of increased bladder pressure B. Persistent low backache from relaxed pelvic joints C. Stronger and more frequent uterine (Braxton Hicks) contractions D. A decline in energy, as the body stores up for labor E. Uterus sinks downward and forward in first-time pregnancies.

After lightening, a return of the frequent need to urinate occurs as the fetal position causes increased pressure on the bladder. In the run-up to labor, women often experience persistent low backache and sacroiliac distress as a result of relaxation of the pelvic joints. Before the onset of labor, it is common for Braxton Hicks contractions to increase in both frequency and strength. Bloody show may be passed. A surge of energy is a phenomenon that is common in the days preceding labor. In first-time pregnancies, the uterus sinks downward and forward about 2 weeks before term. (A, B, C)

Which signs precede the onset of labor? Select all that apply. A. A return of urinary frequency, because of increased bladder pressure B. Persistent low backache, from relaxed pelvic joints C. Stronger and more frequent uterine (Braxton Hicks) contractions D. A decline in energy, as the body stores up for labor E. Weight loss of 0.5 to 1.5 kg

After lightening, a return of the frequent need to urinate occurs, because the fetal position causes increased pressure on the bladder. In the run-up to labor, women often experience persistent low backache and sacroiliac distress as a result of relaxation of the pelvic joints. Prior to the onset of labor it is common for Braxton Hicks contractions to increase in both frequency and strength. Bloody show may be passed and weight loss of 0.5 to 1.5 kg will occur. A surge of energy is a phenomenon that is common in the days preceding labor. (A, B, C, E)

Which statement is inaccurate with regard to normal labor? A. A single fetus presents by vertex. B. It is completed within 8 hours. C. A regular progression of contractions, effacement, dilation, and descent occurs. D. No complications are involved.

Although the amount of time varies with each woman, a normal uncomplicated labor is usually completed within 18 hours. In normal labor, a single fetus presents by vertex. A regular progression of contractions, effacement, dilation, and descent is the trajectory that the nurse expects for a woman experiencing a normal labor, which usually occurs with no complications. (B)

Which laboring client does the nurse expect to be a likely candidate for amnioinfusion? A. A client with heavily meconium-stained amniotic fluid (or a low amniotic fluid index [AFI]) B. A client with an increase in uterine activity C. A client with hypertension and diabetes D. A client with an overdistended uterine cavity

Amnioinfusion is the infusion of room-temperature isotonic fluid into the uterine cavity when the volume of amniotic fluid is low. Clients with premature rupture of membranes are likely to receive an amnioinfusion. The nurse should discontinue the administration of oxytocin for a client with increased uterine activity. Hypertension and diabetes are not factors that indicate the need for amnioinfusion. Pregnant clients with hypertension need to have their blood pressure monitored. Pregnant clients with diabetes need to have their blood glucose levels monitored. Clients receiving amnioinfusion are at a risk for overdistention of the uterine cavity,because amnioinfusion increases the amniotic fluid volume. (A)

Under which circumstance would a nurse not perform a vaginal examination on a patient in labor? A. An admission to the hospital at the start of labor B. When accelerations of the fetal heart rate (FHR) are noted C. On maternal perception of perineal pressure or the urge to bear down D. When membranes rupture

An accelerated FHR is a positive sign not requiring vaginal examination; variable decelerations, however, merit a vaginal examination. Vaginal examination should be performed when the woman is admitted to the hospital or birthing center at the start of labor. When the woman perceives perineal pressure or the urge to bear down is another appropriate time to perform a vaginal examination, as is after rupture of membranes (ROM). The nurse must be aware that there is an increased risk of prolapsed cord immediately after ROM. (B)

*Challenge* Which assessment finding in the client increases the risk for a forceps assisted birth? A. Android pelvis B. Effacement of the cervix C. Biparietal diameter of 9.25 cm D. Involuntary uterine contractions (UCs)

An android pelvis has a narrow subpubic arch and the ischial spines have a narrow interspinous diameter. As a result, the client will have difficulty during a vaginal birth and may require a forceps-assisted delivery. Effacement of the cervix takes place at the onset of the labor and indicates that the client is in labor. A biparietal diameter of 9.25 cm indicates normal fetal head growth, which can be delivered vaginally. Involuntary UCs indicate that the client is in labor. (A)

The nurse is teaching a group of nursing students about fetal oxygenation. The nurse questions a student, "What happens when oxytocin levels are elevated in the client?" What would be the most appropriate answer given by the nursing student related to the client's condition? A. "Hemoglobin levels will decrease." B. "Blood glucose levels will increase." C. "Placenta lowers the blood supply." D. "Uterine contractions (UCs) will increase."

An elevated level of oxytocin increases UCs during labor. Reduced hemoglobin levels lead to a decreased oxygen supply to the fetus, but are not a complication associated with an elevated oxytocin level. Oxytocin has no effect on blood glucose levels. A family history of diabetes may increase the risk of gestational diabetes in the client. Conditions such as hypertension in the client may lower the blood supply to the placenta, but are not associated with oxytocin levels. (D)

Which device can be used as a noninvasive way to assess the fetal heart rate in a client whose membranes are not ruptured? A. Tocotransducer B. Spiral electrode C. Ultrasound transducer D. Intrauterine pressure catheter (IUPC)

An ultrasound transducer is used to assess the FHR by an external mode of electronic fetal monitoring. It does not require membrane rupture and cervical dilation. A tocotransducer can be used to assess the uterine activity (UA) in a pregnant client whose cervix is not sufficiently dilated, but does not assess the FHR. A spiral electrode is used in the internal mode of electronic fetal monitoring to assess the FHR. It can be used only when membranes are ruptured and the cervix is dilated during intrapartum period. IUPC is used to assess the UA in internal mode. It can be used only when membranes are ruptured and the cervix is dilated during the intrapartum period. (C)

*Challenge* The primary health care provider has asked the nurse to draw blood for an umbilical cord acid-base determination test. What should the nurse do in this situation? A. Administer terbutaline (Brethine) before the test. B. Collect blood from both umbilical artery and vein. C. First perform the fetal scalp stimulating technique. D. Only collect blood from the baby's umbilical artery.

An umbilical cord acid-base determination test is performed to assess the immediate condition of the neonate after birth if there is an abnormal or confusing fetal heart rate (FHR) tracing found during the labor. The nurse should collect blood from both umbilical artery and umbilical vein in order to perform the test. Fetal scalp stimulating technique is an indirect method to assess the fetal blood pH. This test need not be performed prior to this acid-base determination test. It is not necessary to administer terbutaline (Brethine) before performing the test; it is administered during the time of labor if uterine contractions (UCs) are too frequent. (B)

What will the nurse mention about the effect of secondary powers during labor to the client? A. Contractions are expulsive in nature. B. The intraabdominal pressure is decreased. C. Contractions move downward in waves. D. Contractions begin at pacemaker points.

As soon as the presenting part of the fetus touches the pelvic floor, the client uses secondary powers or bearing-down efforts. This results in contractions that are expulsive in nature. The voluntary bearing-down efforts of the client also result in increased intraabdominal pressure. Primary powers signal beginning of labor with involuntary contractions that move downward over the uterus in waves. These contractions begin at pacemaker points in the thickened muscle layers of the upper uterine segment. (A)

When evaluating the external fetal monitor tracing of a woman whose labor is being induced, the nurse identifies signs of persistent late deceleration patterns and begins intrauterine resuscitation interventions. Which choice indicates that the following appropriate interventions were implemented in the recommended order of priority? 1. Increase rate of maintenance IV solution. 2. Palpate uterus for tachysystole. 3. Discontinue oxytocin (Pitocin) infusion. 4. Change maternal position to a lateral position, and then elevate her legs if woman is hypotensive. 5. Administer oxygen at 8 to 10 L/minute by nonrebreather face mask. A. 2, 1, 5, 4, 3 B. 4, 1, 2, 3, 5 C. 5, 3, 4, 1, 2 D. 4, 5, 1, 2, 3

B. 4, 1, 2, 3, 5 4. Change maternal position to a lateral position, and then elevate her legs if woman is hypotensive. 1. Increase rate of maintenance IV solution. 2. Palpate uterus for tachysystole. 3. Discontinue oxytocin (Pitocin) infusion. 5. Administer oxygen at 8 to 10 L/minute by nonrebreather face mask.

The nurse explains to a laboring woman that the relaxation periods between contractions are important for which of the following reasons? A. Avoids uterine rupture. B. Allows fetal oxygenation. C. Permits fetal assessment. D. Prevents uterine ischemia

B. Allows fetal oxygenation. (Chapter 12, Pg. 413) Between contractions, the uterus normally returns to a state of complete relaxation. This rest period allows the uterine muscles to relax and provides the woman with a short recovery period that helps her avoid exhaustion. In addition, uterine relaxation between contractions is important for fetal oxygenation because it allows for blood flow from the uterus to the placenta to be restored.

A nurse caring for women in labor should be aware of signs characterizing normal (reassuring) and abnormal (nonreassuring) FHR patterns. What would be characteristics of abnormal patterns? (Circle all that apply) A. Moderate baseline variability. B. Average baseline FHR of 100 beats/minute. C. Acceleration of the FHR with movement. D. Late deceleration patterns approximately every three or four contractions. E. FHR of 170 beats/minute between contractions. F. Early deceleration patterns when the cervix is dilated to 7 cm.

B. Average baseline FHR of 100 beats/minute. D. Late deceleration patterns approximately every three or four contractions. E. FHR of 170 beats/minute between contractions. The baseline should be 110 to 160 beats/minute; accelerations should occur with fetal movement; no late deceleration pattern of any magnitude is normal (reassuring), especially if it is repetitive or uncorrectable; early deceleration patterns are expected findings when fetal head compression by the cervix occurs.

A nulliparous woman is in the active phase of labor and her cervix has progressed to 6 cm dilation. The nurse caring for this woman evaluates the external monitor tracing and notes the following: decrease in FHR shortly after onset of several uterine contractions returning to baseline rate by the end of the contraction; shape is uniform. Based on these findings, the nurse should: A. Change the woman's position to her left side. B. Document the findings on the woman's chart. C. Notify the physician. D. Perform a vaginal examination to check for cord prolapse.

B. Document the findings on the woman's chart. The pattern described is an early deceleration pattern , which is considered to be benign, requiring no action other than documentation of the finding; changing a woman's position and notifying the physician would be appropriate if abnormal (nonreassuring) signs such as late or variable decelerations were occurring; prolapse of cord is associated with variable decelerations as a result of cord compression.

*Challenge* (Not in our Text) Don't use External electronic fetal monitoring will be used for a woman just admitted to the labor unit in active labor. Guidelines that the nurse should follow when implementing this form of monitoring are to: (Circle all that apply) A. Use Leopold maneuvers to determine correct placement of the tocotransducer. B. Assist the woman to maintain a dorsal recumbent position to ensure accurate monitor tracings that can be evaluated easily. C. Apply contact gel to the ultrasound transducer prior to application over the point of maximum intensity of the FHR. D. Reposition the tocotransducer when the fetus changes its position. E. Caution the woman to avoid effleurage (message involving a circular stroking movement with palm) when the transducers are in place. F. Palpate the fundus periodically to estimate the intensity of the uterine contractions.

C and F

Which of the following is considered the primary force of labor? A. Pushing by the mother. B. Uterine contractions. C. Contraction decrement. D. Uterine elongation.

B. Uterine contractions. (Chapter 12, Pg. 413) Uterine contractions are considered the primary force of labor. The use of maternal abdominal muscled for pushing is the secondary force of labor that adds to the primary force to facilitate childbirth. Decrement is the decrease in contraction. Elongation of the uterus is caused by uterine contractions which helps straighten the fetal body.

A woman's amniotic membranes have apparently ruptured. The nurse assesses the fluid to determine its characteristics and confirm membrane rupture. An expected assessment finding of membrane rupture is: A. pH 5.5. B. Absence of ferning. C. Pale, clear fluid with white flecks. D. Strong odor.

C. Pale, clear fluid with white flecks. pH of amniotic fluid is alkaline at 6.5 or higher, ferning is noted when examining fluid with a microscope, and the fluid is relatively odorless; a strong odor is strongly suggestive of infection; white flecks indicate vernix caseosa.

A vaginal examination is performed on a multiparous woman who is in labor. The results of the examination were documented as 4 cm, 75%, +1, ROA. An accurate interpretation of these data is: A. Woman is in the latent phase of the first stage of labor. B. Station is 1 cm above the ischial spines. C. Presentation is cephalic D. Lie is transverse.

C. Presentation is cephalic ("O"), or occiput, indicates a vertex presentation with the neck fully flexed and the occiput in the right anterior pelvic segment (R, A) of the woman's pelvis; the station is 1 cm below the ischial spines (+1); the woman is in the active phase of labor, as indicated by 4 cm of dilation, and effacement is 75%; the lie is longitudinal because the head (cephalic/vertex) is presenting.

A nurse assesses the intensity of a woman's contractions. At the acme of her contraction, the nurse is unable to indent the uterus. How would the nurse document this finding? A. Mild contraction. B. Moderate contraction. C. Strong contraction. D. Intense contraction

C. Strong contraction. (Chapter 12, Pg. 414) When the uterine fundus remains soft at the acme of a contraction, the contraction intensity is described as "mild." Conversely, when there is an inability to indent the uterus at the acme of a contraction, the contraction intensity is described as "strong."

The nurse is caring for a pregnant client whose cervix has dilated to 5 cm and the membranes have ruptured. What assessments does the nurse perform to prevent complications during childbirth? Select all that apply. A. Body temperature every 2 hours B. Fetal heart rate and activity every hour C. Blood pressure and pulse every 30 minutes D. Presence of vaginal show every 15 minutes E. Change in the maternal mood and energy levels every 5 minutes

Cervical dilatation of 5 cm along with ruptured membranes indicates that the client is in the active phase of the first stage of labor. Because the membranes have ruptured, the nurse should assess the client's body temperature every 2 hours, because elevated body temperature may indicate sepsis. Vital parameters such as blood pressure, heart rate, and respiratory rate should be recorded every 30 minutes. Changes in these parameters could indicate cardiopulmonary instability. The nurse should assess for vaginal show every 15 minutes to determine the progress of labor. Fetal heart rate and activity should be assessed every 30 minutes for a client who is in her active phase of first stage of labor. The nurse should monitor the client's mood and energy levels every 15 minutes while the client is in the active phase of first stage of labor. (A, C, D)

*Challenge* The nurse is caring for a Chinese client who is in labor. Which behavior by the client is inconsistent with common Chinese cultural practices? A. The client is extremely anxious during the labor. B. The client opted for natural methods of delivery. C. The client prefers to be in the side-lying position during labor. D. The client does not want the father to be present during labor.

Chinese women are often stoic during labor and are typically able to withstand higher thresholds of pain. Extreme anxiety during labor is not a characteristic behavior of Chinese women. Indian and Japanese women may be more likely to opt for natural delivery than a Chinese woman. Chinese women often prefer being in side-lying position during labor, so this would not be considered an unusual behavior. In Chinese culture, the father is usually not present during labor, so this would not be considered an unusual finding for this client. (A)

When performing vaginal examinations on a laboring woman, the nurse should be guided by what principle? A. Cleanse the vulva and perineum before and after the examination as needed. B. Wear a clean glove lubricated with tap water to reduce discomfort. C. Perform the examination every hour during the active phase of the first stage of labor. D. Perform an examination immediately if active bleeding is present.

Cleansing will reduce the possibility that secretions and microorganisms will ascend into the vagina to the cervix. Maternal comfort will also be enhanced. Sterile gloves and lubricant must be used to prevent infection. Vaginal examinations should be performed only as indicated to limit maternal discomfort and reduce the risk for transmission of infection, especially when rupture of membranes occurs. Examinations are never done by the nurse if vaginal bleeding is present, because the bleeding could be a sign of placenta previa and a vaginal examination could result in further separation of the low-lying placenta. (A)

What are the common signs that are observed in the days preceding labor? Select all that apply. A. Persistent low backache B. Sudden increase in lethargy C. Blood-tinged cervical mucus D. Increase in weight up to 1.5 kg E. Profuse vaginal mucus

Common signs that precede labor include persistent low backache and sacroiliac distress as a result of relaxation of the pelvic joints. Brownish or blood-tinged cervical mucus may be passed. The vaginal mucus becomes more profuse in response to the extreme congestion of the vaginal mucous membranes. In the days preceding labor women generally have a sudden surge of energy. They also experience a loss of 0.5 to 1.5 kg in weight. This is caused by water loss resulting from electrolyte shifts that in turn are produced by changes in estrogen and progesterone levels. (A, C, E)

A laboring woman's uterine contractions are being internally monitored. When evaluating the monitor tracing, which finding is a source of concern and requires further assessment? A. Frequency every 2 1/2 to 3 minutes. B. Duration of 80 to 85 seconds. C. Intensity during uterine contractions of 85 to 90 mm Hg. D. Average resting pressure of 20 to 25 mm Hg.

D. Average resting pressure of 20 to 25 mm Hg (Chapter 12, Pg. 444, Critical Nursing Action: Recognizing and Responding to Uterine Tachysystole) The average resting tone during labor should be 10 mm Hg. Choices A, B, and C are all findings within the expected ranges.

When instructing a group of primigravida women about the onset of labor, the nurse tells them to be alert for: A. Urinary retention. B. Weight gain of 2 kg. C. Quickening. D. Energy surge.

D. Energy surge. (Chapter 12, Pg. 423) Quickening refers to the woman's first perception of fetal movements at 16 to 20 weeks gestation; lightening accompanied by urinary frequency and weight loss of 0.5 to 1 kg occur to signal that the onset of labor is near; backache, stronger Braxton Hicks, and bloody show are also noted.

The perinatal nurse assesses a primigravida who has just arrived at the birth facility for labor assessment. The woman describes contractions that are 7 to 10 minutes apart and felt in the abdomen. She states the contractions feel better when she is walking. This is most likely A. True labor. B. Transition. C. Early labor. D. False labor.

D. False Labor (Chapter 12, Pg. 425, Table 12-1 Distinguishing True from False labor) False labor: Contractions are irregular. Usually there is no increase in frequency, duration, or intensity of contractions. Pains usually occur in the abdominal region. There is no change in the cervix. Walking may lessen the pain. True labor: Contractions are at regular intervals. Contractions increase in frequency, duration, and intensity. Pains usually begin in lower back, radiating to the abdomen. Dilation and effacement of the cervix are progressive. Activity such as walking usually increases labor pains.

A nurse assesses a woman in labor and finds that her contractions are occurring once every 1 to 1 and one-half minutes with a uterine resting tone greater than 30 mm Hg. How does the nurse document this finding? A. First stage labor, active phase. B. First stage labor, transition. C. Second stage labor. D. Uterine tachysystole.

D. Uterine tachysystole. (Chapter 12, Pg. 444, Critical Nursing Action: Recognizing and Responding to Uterine Tachysystole) Sometimes referred to as hyperstimulation, uterine tachysystole is a condition of excessively frequent contractions with the following characteristics: 5 uterine contractions in 10 minutes, with contractions occuring within 1 minute of each other, and a uterine resting tone greater than 20 to 25 mm Hg with a peak pressure greater than 80 mm HG.

What finding in a client indicates a potential risk for complications during the labor process? A. Maternal temperature of 99.7 F B. Persistent dark red vaginal bleeding C. Intrauterine pressure of 50 mm Hg D. Contractions lasting for 70 seconds

Dark red blood is indicative of an old uterine bleed which was left untreated. It may indicate fetal hypoxia. Therefore, a persistent flow of dark red vaginal bleeding is a sign of a potential complication during the process of labor. A maternal body temperature of 99.7o F is normal and does not indicate any complication. Intrauterine pressure greater than 80 mm Hg is a sign of potential complications. Contractions lasting for more than 90 seconds may increase risk during labor. (B)

What are the factors that speed up the dilation of the cervix? Select all that apply. A. Strong uterine contractions B. Scarring of the cervix C. Pressure by amniotic fluid D. Prior infection of the cervix E. Force by fetal presenting part

Dilation of the cervix occurs by the drawing upward of the musculofibrous components of the cervix, which are in turn caused by strong uterine contractions. Pressure exerted by the amniotic fluid while the membranes are intact or by the force applied by the presenting part can promote cervical dilation. Scarring of the cervix may occur following a surgery. Prior infection or surgery may slow cervical dilation. (A, C, E)

The nurse instructs a pregnant client to breathe through the mouth and keep it open while pushing during labor. What is the rationale for this nursing intervention? A. Avoiding nasal congestion in the client B. To decrease efforts required for pushing C. Facilitating increased oxygen to the fetus D. To avoid deceleration in fetal heart rate

During labor the nurse asks the client to breathe through the mouth to keep her mouth open to increase both maternal and fetal oxygenation. Nasal congestion is not a complication associated with labor. Opening of the mouth does not increase the pushing capability. Early decelerations are observed by pushing, which does not require any intervention. (C)

The nurse knows that what occurs in the second stage of labor, the descent phase? A. The amniotic membranes rupture. B. The cervix cannot be felt during a vaginal examination. C. The woman experiences a strong urge to bear down. D. The presenting part is below the ischial spines.

During the descent phase of the second stage of labor, the woman may experience an increase in the urge to bear down. Rupture of membranes has no significance in determining the stage of labor. The second stage of labor begins with full cervical dilation. Many women may have an urge to bear down when the presenting part is below the level of the ischial spines. This can occur during the first stage of labor, as early as 5 cm of dilation. (C)

Which parameter should be closely monitored in a patient during the latent phase of the first stage of labor? A. Fetal heart rate B. Cervical dilation C. Maternal temperature D. External cephalic version

During the first stage of labor, uterine contractions have just begun and the fetus is monitored for various parameters. The fetal heart rate is monitored atleast every 30 to 60 minutes to ensure the safety of the fetus. Cervical dilation is assessed through vaginal examination and helps to determine the approximate time required for delivery. Maternal temperature is monitored every 2 to 4 hours to ensure the patient's safety. External cephalic version is performed to align the fetus to the birth canal. (A)

While assessing the fundus of a postpartum client, the nurse places both hands, one over the other, on the client's abdomen and applies a downward pressure toward the vagina. What are the rationales behind this intervention? Select all that apply. A. To assess laceration repair for redness B. To help the client to void spontaneously C. To determine whether the fundus is firm D. To assist the client in expelling clots E. To assess for the presence of hemorrhoids

During the fourth stage of labor, the nurse must assess parameters such as blood pressure, temperature, and tone of the fundus. The tone of the fundus is assessed to determine if the client is at risk of having a postpartum hemorrhage. Massaging the fundus also helps to expel retained clots from the vagina. Redness or edema is observed while caring for lacerations; however, applying pressure toward the vagina does not aid in repairing the lacerations. The nurse helps the client to void spontaneously to help the client urinate. The perineum is assessed for the presence of hemorrhoids, but applying downward pressure toward the vagina would not be helpful in assessing hemorrhoids.(C, D)

What happens during the second stage of labor? Select all that apply. A. The second stage of labor is the stage in which the infant is born. B. In this stage, the birthing table is usually set up for the nulliparous woman. C. The progress of labor is enhanced when a woman changes her position frequently during this stage of labor. D. This stage begins with full cervical dilation (10 cm) and complete effacement (100%) and ends with the baby's birth. E. The force exerted by uterine contractions, gravity, and maternal bearing-down efforts facilitates achievement of the expected outcome of a spontaneous, uncomplicated vaginal birth. F. The median duration of this stage of labor is 50 to 60 minutes in nulliparous women and 20 to 30 minutes in multiparous women.

During the second stage of labor the infant is born; this stage begins with full cervical dilation (10 cm) and complete effacement (100%) and ends with the baby's birth. The force exerted by uterine contractions, gravity, and maternal bearing-down efforts facilitates achievement of the expected outcome of a spontaneous, uncomplicated vaginal birth. The median duration of second-stage labor is 50 to 60 minutes in nulliparous women and 20 to 30 minutes in multiparous women. In the first stage of labor, the birthing table is usually not set up for the nulliparous woman, and the progress of labor is enhanced when a woman changes her position frequently. (A, C, D, E, F)

The nurse says, "You are doing so well; do it again" to a client during the second stage of labor. Why did the nurse say this? A. To promote comfort and minimize distractions B. To promote bearing-down efforts in the client C. To encourage the client to feel confident D. To promote adequate oxygen levels in the fetus

During the second stage of labor, the client experiences severe pain, fear, anxiety, and confusion. The client might scream during the active pushing stage. Therefore, nurse encourages the client to feel confident in her body. The nurse dims the lights during labor and speaks quietly in order to comfort the client and to minimize distractions. Once the client gains confidence, the bearing-down effort improves. Adequate oxygen levels in the maternal blood can be maintained by asking the client to take rapid breaths. (C)

Which finding does the nurse relate to normal uterine activity in the second stage of labor? A. Contraction frequency is three in 10 minutes. B. Contractions peak at 30 to 50 mm Hg. C. Strength of the contraction is over 80 mm Hg. D. Relaxation time is 30 seconds or less.

During the second stage of labor, the strength of the contractions may increase to over 80 mm Hg. The frequency of contractions is normally up to five contractions every 10 minutes during the second stage of labor. In the first stage, the frequency ranges from two to five contractions in 10 minutes. The contractions peak at 40 to 70 mm Hg in the first stage of labor. The relaxation time is 60 seconds or more during the first stage of labor. This can reduce to 45 seconds or more in the second stage, but should not drop as low as 30 seconds because of risk of fetal distress. (C)

The nurse is assisting the primary health care provider during a client's vaginal delivery. Which nursing intervention is performed to reduce the extent of vaginal or perineal lacerations? A. Placing clean material under the client's buttocks B. Touching the client's vaginal area to promote birth C. Applying gentle pressure toward the client's vagina D. Allowing rapid birth of the fetal head

During vaginal delivery, the client may sustain deep vaginal and perineal lacerations. Though lacerations cannot be prevented during fetal birth, the extent of the lacerations can be reduced. Fetal birth occurs as the fetal head is rapidly expelled from the vagina. The pressure produced during the sudden fetal birth results in deep vaginal and perineal lacerations. Therefore, the flat side of the hand is placed on the exposed fetal head and gentle pressure is applied toward the vagina. This prevents the fetal head for popping out, and prevents lacerations. The nurse should place clean material under the client's buttocks to reduce the soiling of linen due to vaginal bleeding during birth. The client's vaginal area should not be touched during birth in order to avoid infection and provide privacy to the client. Rapid birth of the fetal head causes a rapid change of pressure within the molded fetal skull, which could result in dural or subdural tears. Therefore, this intervention does not aid in reducing lacerations. (C)

While performing a vaginal examination of the client in active labor, the nurse notes decelerations in the fetal heart during uterine contractions. What should the nurse do in this situation? A. Stop applying fundal pressure. B. Discontinue the oxytocin (Pitocin) drip. C. Change the maternal position. D. Document it as a normal finding.

Early decelerations in the FHR are common during UCs. These decelerations may also happen due to increased fundal pressure and during placement of the internal mode of fetal monitoring. Applying fundal pressure during vaginal examination helps in accurate assessment; therefore, fundal pressure needs to be applied. Oxytocin (Pitocin) administration induces UCs and indirectly causes the FHR to decelerate, but the medication should not be discontinued. The nurse would have the client change positions if the umbilical cord was compressed. (D)

A group of fetal monitoring experts (National Institute of Child Health and Human Development, 2008) recommends that fetal heart rate (FHR) tracings demonstrate certain characteristics to be described as reassuring or normal (category I). These characteristics include: A. Bradycardia not accompanied by baseline variability. B. Early decelerations, either present or absent. C. Sinusoidal pattern. D. Tachycardia.

Early decelerations, the absence of late decelerations, and the presence of accelerations indicate a normal category I tracing. Bradycardia not accompanied by variability is a category II tracing, as is fetal tachycardia. A sinusoidal pattern is considered an ominous sign and is definitely an abnormal category III tracing. (B)

Which test is performed to determine if membranes are ruptured? A. Urine analysis B. Fern test C. Leopold maneuvers D. AROM

In many instances a sterile speculum examination and a Nitrazine (pH) and fern test are performed to confirm that fluid seepage is indeed amniotic fluid. A urine analysis should be performed on admission to labor and delivery. This test is used to identify the presence of glucose and protein. The nurse performs Leopold maneuvers to identify fetal lie, presenting part and attitude. AROM is the procedure of artificially rupturing membranes usually with a device known as an amnihook. (B)

*Challenge* After performing the Leopold's maneuver on a client at 38 weeks of pregnancy, the nurse concludes that the client will require external cephalic version for having a vaginal delivery. What was the finding during assessment? A. The client had a short pelvic outlet. B. The fetus is in the breech presentation. C. The fetus had not descended in the pelvis. D. The client has weak uterine contractions.

External cephalic version is an ultrasound-guided, hands-on technique that is used to externally manipulate the fetus into a cephalic lie. This technique is indicated when the fetus has a noncephalic presentation such as breech presentation. If the client has a short pelvic outlet, the client must usually undergo a caesarean delivery. External cephalic version cannot be done if the fetus has not descended in the pelvis. The client is not in labor, so the client will have weak uterine contractions. (B)

*DELETE* The sonographic reports of a pregnant client reveal extreme asynclitism of the fetal head. What does the nurse conclude from this report? A. The fetal head is parallel to the anteroposterior plane of the pelvis. B.The client will have a normal vaginal delivery. C. The position of the fetal head will facilitate descent. D. Cephalopelvic disproportion will be seen during labor.

Extreme asynclitism of the fetal head makes the fetus unable to descend during the birth process and causes cephalopelvic disproportion. The fetal head is parallel to the anteroposterior plane of the pelvis in a synclitic position. The client will most probably have a cesearean delivery because extreme asynclitism indicates that the fetal head is deflected in a way that may interfere with vaginal delivery. Asynclitism, not extreme asynclitism, facilitates fetal descent, because the head is being positioned to accommodate the pelvic cavity. (D)

When assisting a client in labor, the nurse expects to observe the cardinal movements that lead to the birth of the baby. Arrange the movements in the order of their occurrence. A. Internal rotation B. Flexion C. Extension D. Descent E. Restitution F. Engagement

F. Engagement D. Descent B. Flexion A. Internal rotation C. Extension E. Restitution The cardinal movements that occur in a vertex presentation are engagement, descent, flexion, internal rotation, extension, restitution (external rotation), and finally birth by expulsion. The fetal head is said to be engaged in the pelvic inlet, when the biparietal diameter of the head passes through the pelvic inlet. During descent, the presenting part progresses through the pelvis. As soon as the descending head meets resistance from the cervix or pelvic wall or pelvic floor, it undergoes flexion. The fetus flexes such that the chin is brought into closer contact with the fetal chest. Internal rotation begins at the level of the ischial spines but is not completed until the presenting part reaches the lower pelvis. When the fetal head reaches the perineum for birth, it undergoes extension. It is deflected anteriorly by the perineum. Restitution occurs after the head is born. It rotates briefly to the position it occupied when it was engaged in the inlet.

The nurse has performed vibroacoustic stimulation and determines that the fetal heart rate has increased by 15 beats/min from the baseline within 15 seconds. Which condition does this acceleration indicate? A. Mixed acidemia in the fetus B. Signs of respiratory acidemia C. A normal pH level in the fetus D. Elevated pCO2 level in the fetus

FHR acceleration by about 15 beats/min in 15 seconds on vibroacoustic stimulation indicates a normal blood pH of the fetus. A decreased pH and elevated carbon dioxide pressure indicate respiratory acidemia. Therefore FHR acceleration is not indicative of either respiratory acidemia or mixed acidemia in the fetus. The umbilical cord acid-base method involves the determination of both the carbon dioxide pressure and oxygen pressure of the fetus. (C)

*DELETE DUPLICATE* Fetal bradycardia is common during what problem? A. Maternal hyperthyroidism B. Fetal anemia C. Viral infection D. Tocolytic treatment using ritodrine

Fetal bradycardia can be considered a later sign of fetal hypoxia and is known to occur before fetal death. Bradycardia can result from placental transfer of drugs, viral infections such as cytomegalovirus (CMV), maternal hypothermia, and maternal hypothermia. Maternal hyperthyroidism will most likely result in fetal tachycardia. Fetal anemia will most likely result in fetal tachycardia. Tocolytic treatment using ritodrine will most likely result in fetal tachycardia. (C)

What instructions does the nurse give a client to ensure fetal safety during the second stage of labor? A. "Push continuously on command." B. "Hold your breath and push." C. "Push when you feel the urge." D. "Hold your breath and push with maximum effort."

Fetal safety depends on the client's breathing patterns during the labor process, and on her bearing-down efforts. The nurse should ask the client to push instinctively and spontaneously to ensure more effective bearing-down efforts. This ensures the safety of the fetus by preventing hypoxia. The nurse should not ask the client to push continuously on command, because it may result in the client closing her glottis. The nurse should not ask the client to hold her breath and push, because it may increase intrathoracic and cardiovascular pressure. This would ultimately lead to fetal hypoxia. Telling the client to hold her breath and push with maximum effort before she feels the urge to push down is also discouraged, because this reduces cardiac output and decreases perfusion of the uterus and the placenta. (C)

Fetal well-being during labor is assessed by: A. The response of the fetal heart rate (FHR) to uterine contractions (UCs). B. Maternal pain control. C. Accelerations in the FHR. D. An FHR greater than 110 beats/min.

Fetal well-being during labor can be measured by the response of the FHR to UCs. In general, reassuring FHR patterns are characterized by an FHR baseline in the range of 110 to 160 beats/min with no periodic changes, a moderate baseline variability, and accelerations with fetal movement. Although FHR accelerations and an FHR greater than 110 beats/min may be reassuring, they are only two components of the criteria by which fetal well-being is assessed. More information is needed to determine fetal well-being. (A)

Fetal well-being during labor is assessed by monitoring what? A. The response of the fetal heart rate (FHR) to uterine contractions (UCs) B. Maternal pain control C. Accelerations in the FHR D. An FHR greater than 110 beats/minute

Fetal well-being during labor can be measured by the response of the FHR to UCs. In general, reassuring FHR patterns are characterized by an FHR baseline in the range of 110 to 160 beats/minute with no periodic changes, a moderate baseline variability, and accelerations with fetal movement. Maternal pain control is not the measure used to determine fetal well-being in labor. Although FHR accelerations are a reassuring pattern, they are only one component of the criteria by which fetal well-being is assessed. Although an FHR greater than 110 beats/minute may be reassuring, it is only one component of the criteria by which fetal well-being is assessed. More information is needed to determine fetal well-being. (A)

A client complains of the urge to have a bowel movement during each contraction. What does the nurse inform the client ? A. "There is a possibility of an infection." B. "I will have to evaluate your urine reports." C. "There is a complication with the delivery." D. "This is a normal occurrence at the onset of labor."

Frequent bowel movements may be seen in some clients at the onset of labor because of the presence of stool in the rectum. Therefore, the nurse should instruct the client that it is a normal occurrence. Bowel movement during each contraction does not indicate an infection, so there is no need to evaluate the urine reports. The possibility of a complication during delivery can be confirmed only after evaluating the ultrasound reports of the client. (D)

Which statement is accurate with regard to normal labor? Select all that apply. A. A single fetus presents by vertex. B. It is completed within 8 hours. C. A regular progression of contractions, effacement, dilation, and descent occurs. D. No complications are involved. E. Mechanisms of labor are involved.

In normal labor, a single fetus presents by vertex. A regular progression of contractions, effacement, dilation, and descent is the trajectory that the nurse expects for a woman experiencing a normal labor. A normal labor usually has no complications and the movements of the mechanisms of labor are present. Although the amount of time varies with each woman, a normal uncomplicated labor is usually completed within 18 hours. (A, C, D, E)

In a variation of rooming-in called couplet care, the mother and infant share a room and the mother shares the care of the infant with: A. The father of the infant. B. Her mother (the infant's grandmother). C. Her eldest daughter (the infant's sister). D. The nurse.

In couplet care the mother shares a room with the newborn and shares infant care with a nurse educated in maternity and infant care. This may also be known as mother-baby care or single-room maternity care. The father is included in instruction regarding infant care whenever he is present. The grandmother is welcome to stay and take part in the woman's postpartum care, but she is not part of the couplet. An older sibling may stay with the client and her baby but is also not part of the couplet. (D)

During the vaginal examination of a client, the nurse notes that the fetus is in an oblique lie. What does this indicate? A. The fetus cannot be born by vaginal birth. B. The long axis of the mother and fetus are parallel. C. The presentation is either cephalic or breech. D. The fetal lie will undergo change during labor

If the fetus is in an oblique lie, it usually converts to a longitudinal or transverse lie during labor. In an oblique lie the long axis of the fetus lies at an angle to the long axis of the mother. Fetal lie is the relation of the long axis (spine) of the fetus to the long axis (spine) of the mother. Vaginal birth cannot occur when the fetus stays in a transverse lie. In the longitudinal lie, the long axis of the fetus is parallel with the long axis of the mother. Longitudinal lies are either cephalic or breech presentations, depending on the fetal structure that first enters the mother's pelvis. (D)

A pregnant client who is nearing her due date informs the nurse that she would like a vaginal delivery. The nurse observes in the medical records that the presenting part is the sacrum. What does the nurse tell the client? A. "Vaginal delivery may not be possible." B. "There will be no complications during labor." C. "You may have to lose weight for a safe delivery." D. "The infant may have congenital physical defects."

If the presenting part of the fetus is the sacrum, it indicates a breech presentation. Vaginal delivery of a fetus in breech position carries increased risks and it is more likely that the client will have to have a caesarean delivery. It is inaccurate to inform the client that there will be no complications during the birth, because this is not something that the nurse can predict. A breech presentation does not indicate that the client needs to lose weight. Environmental and biologic factors are associated with congenital defects. (A)

The nurse is monitoring the fetal status of a client in labor. What are the causes that can lead to a decrease in fetal oxygen supply? Select all that apply. A. Maternal obesity B. Umbilical cord prolapse C. Supine maternal position D. Increase in blood volume E. Excessive exogenous oxytocin

If the umbilical cord prolapses, it can decrease the amount of oxygenated blood to the fetus. A supine maternal position can lead to hypotension, which reduces the blood flow through the maternal vessels and in turn decreases the fetal oxygen supply. Excessive exogenous oxytocin production can lead to uterine hypertonus. This reduces the blood flow to the intervillous space in the placenta. Maternal obesity does not decrease fetal blood flow. Hemorrhage or anemia leading to decrease in blood volume can cause a decrease in the fetal oxygen supply. An increase in blood volume would increase oxygen levels. (B, C, E)

Which test is performed to determine whether membranes are ruptured? A. Urine analysis B. Fern test C. Leopold maneuvers D. AROM

In many instances a sterile speculum examination and a Nitrazine (pH) and fern test are performed to confirm that fluid seepage is indeed amniotic fluid. A urine analysis should be performed on admission to labor and delivery to determine the presence or absence of glucose and protein. The nurse performs Leopold maneuvers to identify fetal lie, presenting part, and attitude. AROM is the procedure of artificially rupturing membranes, usually with a device known as an amnihook. (B)

The nurse notes that the fetus in a laboring patient is in brow presentation. What is the expected occipitomental diameter? A. 9.25 cm B. 9.5 cm C. 12.0 cm D. 13.5 cm

In a brow presentation, the presenting part is the mentum or chin. The occipitomental diameter is 13.5 cm at term, which is too large to permit the infant's head to enter the pelvis region of the mother. The biparietal diameter, which is about 9.25 cm at term, is the largest transverse diameter. The smallest anteroposterior diameteris, the suboccipitobregmatic diameter, about 9.5 cm at term, is in a vertex presentation. In a sinciput presentation, the occipitofrontal diameter is about 12.0 cm at term, with moderate extension of the head. (D)

Challenge *Not in our Text* The sonographic reports of a client indicate that the fetus has a cephalic presentation. What does the nurse tell the client? A. "The fetal skull is not flexible." B. "There are fetal complications." C. "You may need a cesarean delivery." D. "Vaginal delivery will be the best choice."

In a cephalic presentation, the fetal head may be flexed and exceed the limits of the maternal pelvis. Therefore, a vaginal birth may have complications and the client may have a forceps or vacuum-assisted birth or cesarean birth. The fetal skull is flexible due to the open fontanels during birth. Fetal complications may be seen during labor if there is a forceps or vacuum-assisted birth, which may cause birth trauma. Therefore, vaginal delivery is not the best choice in this case. (C)

The nurse is assisting a client during the active phase of the second stage of labor. The nurse finds that the client has a sudden, significant increase in dark red bloody show. What does the nurse infer from this observation? A. The discharge indicates that the fetal head is about to deliver. B. The discharge indicates high maternal blood pressure. C. The discharge indicates a potential complication of labor. D. The discharge indicates maternal and fetal infection.

In a normal vaginal delivery, the fetus head is born first. The passage for childbirth gets stretched as the fetal head descends down, resulting in increased dark red bloody show. Therefore, a significant increase in dark red bloody show indicates the birth of the fetal head. The dark red colored bleeding does not indicate high blood pressure. A significant increase in bleeding is a potential complication in the fourth stage of labor. The seepage of amniotic fluid may indicate that the amniotic sac had broken, which may lead to maternal and fetal infection. (A)

In order to accurately assess the health of the mother accurately during labor, the nurse should be aware that: A. The woman's blood pressure increases during contractions and falls back to prelabor normal between contractions. B. Use of the Valsalva maneuver is encouraged during the second stage of labor to relieve fetal hypoxia. C. Having the woman point her toes reduces leg cramps. D. The endogenous endorphins released during labor raise the woman's pain threshold and produce sedation.

In addition, physiologic anesthesia of the perineal tissues, caused by the pressure of the presenting part, decreases the mother's perception of pain. Blood pressure increases during contractions but remains somewhat elevated between them. Use of the Valsalva maneuver is discouraged during second-stage labor because of a number of unhealthy outcomes, including fetal hypoxia. Pointing the toes can cause leg cramps, as can the process of labor itself. (D)

After observing the reports of umbilical cord acid-base determination test, the nurse informs the client that the newborn's condition is normal. Which value indicates the normal condition of the newborn? A. Umbilical artery: pH -7.1, Pco2 -50 mm Hg, Po2 -20 mm Hg B. Umbilical artery: pH -7.3, Pco2 -40 mm Hg, Po2 -10 mm Hg C. Umbilical artery: pH -7.4, Pco2 -52 mm Hg, Po2 -27 mm Hg D. Umbilical artery: pH -7.3, Pco2 -45 mm Hg, Po2 -25 mm Hg

In umbilical cord acid-base stimulation method, arterial values indicate the condition of the newborn. Arterial blood pH of 7.2 to 7.3, carbon dioxide pressure (Pco2) value of 45 to 55 mm Hg, and oxygen pressure (Po2) value of 15 to 25 mm Hg approximately indicates the normal fetal condition. Therefore pH: 7.3, Pco2 -45, Po2 -25 values are representing the normal fetal condition. Arterial blood pH -7.1, Pco2 -50 mm Hg, Po2 -20 mm Hg indicate that the fetus may have respiratory acidosis. Arterial blood pH -7.4 is indicative of fetal alkalosis. (D)

Which are the factors that affect the onset of labor? Select all that apply. A. Increasing intrauterine pressure B. Increasing estrogen levels C. Decreasing oxytocin levels D. Decreasing progesterone levels E. Decreasing prostaglandin levels

Increasing intrauterine pressure, increasing estrogen levels, and decreasing progesterone levels affect the onset of labor. Increasing intrauterine pressure is associated with increasing myometrial irritability. This is caused by increasing concentrations of estrogen and decreasing progesterone levels. Oxytocin and prostaglandin levels are known to increase during the onset of labor. (A, B, D)

The nurse observes late decelerations of the fetal heart rate (FHR) in the second phase of labor of a pregnant patient. The nurse assesses the pregnant patient and elevates the lower extremities of the patient. Which assessment finding would be the reason for this nursing intervention? A. Placental abruption B. Maternal hypotension C. Maternal hemorrhage D. Uterine contractions

Late decelerations in the FHR may be caused by maternal hypotension. Elevating the lower extremities helps to control maternal hypotension and increase the blood flow to the uterus. Elevating the legs wound not control hemorrhage, placental abruption, or UCs in a pregnant patient. (B)

The nurse is teaching a client, who is pregnant for the first time, about the signals that indicate the beginning of labor. Which sign will the nurse mention as a signal for the beginning of labor? A. Involuntary contractions B. Pain in the pelvic joints C. 100% effacement of the cervix D. Full dilation of the cervix

Involuntary uterine contractions, or the primary powers, signal the beginning of labor. Pain in the pelvic joints does not signal the beginning of labor. It is a result of widening of the joint of the symphysis pubis and the resulting instability. The primary powers are responsible for the effacement and dilation of the cervix and the descent of the fetus. Effacement is the shortening and thinning of the cervix during the first stage of the labor. However, 100% effacement would indicate that the patient is well established in the labor process. Dilation of the cervix is the enlargement or widening of the cervical opening and cervical canal. This dilation progresses after the labor has begun. Full cervical dilation marks the end of the first stage of labor. (A)

Which position would the nurse suggest for second-stage labor if the pelvic outlet needs to be increased? A. Semirecumbent B. Sitting C. Squatting D. Side-lying

Kneeling or squatting moves the uterus forward and aligns the fetus with the pelvic inlet; this can facilitate the second stage of labor by increasing the pelvic outlet. A semirecumbent position does not assist in increasing the size of the pelvic outlet. Although sitting may assist with fetal descent, this position does not increase the size of the pelvic outlet. A side-lying position is unlikely to assist in increasing the size of the pelvic outlet. (C)

Which of the following would not be included in a labor nurse's plan of care for an expectant mother? A. The onset of progressive, regular contractions B. The bloody, or pink, show C. The spontaneous rupture of membranes D. Formulation of the woman's plan of care for labor

Labor care begins when progressive, regular contractions begin, the blood-tinged mucoid vaginal discharge appears, or fluid is discharged from the vagina. The woman and the nurse can formulate their plan of care before labor or during treatment. (D)

The nurse is performing Leopold's maneuver in a client who is in the first stage of labor. What information does the nurse obtain while performing these maneuvers? Select all that apply. A. The fetal heart rate B. The fetal part in the fundus C. The tone of the uterus D. The presenting part of the fetus E. The descent of the fetus into the pelvis

Leopold maneuvers, or abdominal palpation, during first stage of labor help determine the fetal part present in the fundus, which indicates the fetal lie. The presenting part of the fetus would help determine if the patient should undergo vaginal birth or requires a cesarean delivery. The position and location of the fetal back helps to determine the descent of the fetus into the pelvis, which indicates the approximate time required for vaginal delivery. The fetal heart rate can only be auscultated using a Doppler ultrasound. However, using abdominal palpation, the point of maximum intensity of the fetal heart rate can be determined. The nurse may not assess the tone of the fundus during the first stage of labor. The tone of the fundus is assessed after the delivery of the child to determine the risk of postpartum hemorrhage. (B, D, E)

Fetal circulation can be affected by many factors during labor. Accurate assessment of the laboring woman and fetus requires knowledge of these expected adaptations. Which factor will not affect fetal circulation during labor? A. Fetal position B. Uterine contractions C. Blood pressure D. Umbilical cord blood flow

Maternal position may affect fetal circulation; however, fetal position is unlikely to disturb umbilical blood flow. Uterine contractions during labor tend to decrease circulation and subsequent perfusion. Most healthy fetuses are well able to compensate for this stress and exposure to increased pressure while moving passively through the birth canal during labor. Maternal blood pressure is likely to have a significant effect on fetal circulation. Compression of the cord and reduction of umbilical blood flow do affect fetal circulation. (A)

The nurse caring for a laboring woman is aware that maternal cardiac output can be increased by what? A. Change in position B. Oxytocin administration C. Regional anesthesia D. Intravenous analgesic

Maternal supine hypotension syndrome is caused by the weight and pressure of the gravid uterus on the ascending vena cava when the woman is in a supine position. This reduces venous return to the woman's heart, as well as cardiac output, and subsequently reduces her blood pressure. The nurse can encourage the woman to change positions and avoid the supine position. Oxytocin administration may reduce maternal cardiac output. Regional anesthesia may reduce maternal cardiac output. Intravenous analgesic may reduce maternal cardiac output. (A)

After monitoring the fetal heart activity the nurse documents the fetal heart rate (FHR) to be in category II, according to the three-tier FHR classification system. What findings would the nurse have observed? A. Minimal variability. B. Moderate variability. C. Less than 110 beats/minute. D. Accelerations were present.

Minimum variability in the FHR indicates that there is insufficient oxygen supply to the fetus. This is categorized as a category II in a three-tiered FHR classification system. Moderate variability in FHR indicates the normal cardiac activity of the fetus, which is categorized under category I. Bradycardia (FHR less than 110 beats/minute) is categorized under category III. The FHR acceleration is completely absent according to category II and is present in category I. (A)

While assessing the fetal heart rate (FHR) of a client in labor, what does the nurse identify as normal variability of the FHR? A. Absent variability B. Minimal variability C. Moderate variability D. Marked variability

Moderate variability is highly predictive of a normal fetal acid-base balance. It indicates that FHR regulation is not significantly affected by fetal sleep cycles, tachycardia, prematurity, congenital anomalies, preexisting neurologic injury, or central nervous system (CNS) depressant medications. Absent or minimal variability is classified as either abnormal or indeterminate. It can result from fetal hypoxemia and metabolic academia. The significance of marked variability is unclear. (C)

*Challenge* The charge nurse instructed a group of student nurses about monitoring of uterine activity during labor. Which statement by the student nurse is accurate regarding the calculation of Montevideo units? (hint: internal monitoring) A. They can be calculated by using a spiral electrode monitoring device. B. They can be calculated by using a tocotransducer monitoring system. C. They can be calculated by utilizing an ultrasound transducer machine. D. They can be calculated with an intrauterine pressure catheter (IUPC).

Montevideo units can only be calculated using the internal monitoring of UA. An intrauterine pressure catheter (IUPC) monitors UA internally. Therefore, Montevideo units can only be calculated using the IUPC. Spiral electrode monitoring is used for assessing the fetal heart rate (FHR), not UA internally. The tocotransducer monitoring system is used to monitor the UA externally. An ultrasound transducer is also used to monitor the FHR externally. (D)

The diagnostic test reports of a pregnant client reveal a baseline fetal heart rate of 175 beats/minute. What does this finding indicate to the nurse? A. Presence of fetal ischemia. B. Fetal tachycardia. C. Fetal bradycardia. D. The fetus has hypotension.

Normal baseline fetal heart rate ranges from 110 to 160 beats/minute. If the fetal heart rate is more than 160 beats/minute, then tachycardia in the fetus is indicated. Ischemia is a condition in which there is a reduced blood supply to the fetal tissues. Baseline heart rate below 110 beats/minute indicates bradycardia in fetus. Hypotension indicates a blood pressure level below 120/80 mm Hg, which is a life-threatening condition for the fetus. (B)

A primigravida asks the nurse about signs she can look for that indicate that the onset of labor is getting closer. The nurse should describe what? A. Weight gain of 1 to 3 lbs. B. Quickening. C. Fatigue and lethargy. D. Bloody show.

Passage of the mucus plug (operculum) also termed pink/bloody show occurs as the cervix ripens. Women usually experience a weight loss of 1 to 3 lbs. Quickening is the perception of fetal movement by the mother, which occurs at 16 to 20 weeks of gestation. Women usually experience a burst of energy or the nesting instinct. (D)

If a woman complains of back labor pain, what is the best suggestion by the nurse? A. Lie on her back for a while with her knees bent B. Do less walking around C. Take some deep, cleansing breaths D. Lean over a birth ball with her knees on the floor

The hands-and-knees position, with or without the aid of a birth ball, should help with the back pain. The supine position should be discouraged. Walking generally is encouraged. Deep cleansing breaths will assist with any labor pain; however, it is very important that this woman's position is changed so that she is not on her back. (D)

When assessing a multiparous woman who has just given birth to an 8-pound boy, the nurse notes that the woman's fundus is firm and has become globular. A gush of dark red blood comes from her vagina. The nurse concludes that: A. The placenta has separated. B. A cervical tear occurred during the birth. C. The woman is beginning to hemorrhage. D. Clots have formed in the upper uterine segment.

Placental separation is indicated by a firmly contracting uterus, a change in the uterus from a discoid to a globular ovoid shape, a sudden gush of dark red blood from the introitus, an apparent lengthening of the umbilical cord, and a finding of vaginal fullness. Cervical tears that do not extend to the vagina result in minimal blood loss. Signs of hemorrhage are a boggy uterus, bright red vaginal bleeding, alterations in vital signs, pallor, lightheadedness, restlessness, decreased urinary output, and alteration in the level of consciousness. If clots have formed in the upper uterine segment, the nurse would expect to find the uterus boggy and displaced to the side. (A)

A pregnant client who is full term has a cervical dilatation of 2 cm. The nurse asks the client to get admitted the next day, but after talking to the client, the nurse allows the client to be admitted the same day. What are the reasons for admitting the client in the latent stage of labor? Select all that apply. The client lives far away from the birth center. The client has occasional strong uterine contractions. The client had rapid labors in the past two deliveries. The client is a single mother and has no other family. The client is stressed and anxious regarding the birthing process.

Pregnant women who are in labor are generally not admitted to the birth center until the cervix is dilated to 3 cm. In this case, the client's cervical dilatation is only 2 cm but the client may be admitted early if the client lives far from the birth center, making it difficult for the client to travel back when her labor progresses. Early admission may be particularly important if a client has had rapid births in the past because it's likely she will have another rapid labor this time. Early admission may also be allowed if the client lacks family support, such as in the case of a single mother. This client is in the early phase of labor and, therefore, is expected to have occasional strong uterine contractions. This is not a criterion for admitting the client. Stress and anxiety regarding the birthing process is common in clients who are in labor. This is not a criterion for early admission to the birthing center. (A, C, D)

Which fetal presentations can be seen during birth? Select all that apply. A. Breech B. Oblique C. Cephalic D. Shoulder E. Transverse

Presentation refers to the part of the fetus that enters the pelvic inlet first and leads through the birth canal during labor. The breech presentation means that the buttocks, feet, or both the buttocks and feet together will appear first. The cephalic presentation means that the fetal head will be the first part to appear through the birth canal. The shoulder presentation means that the presenting part is the scapula. The terms oblique and transverse refer to the fetal lie, in which the long axis of the fetus is at a right angle diagonal to the long axis of the mother. (A, C, D)

The nurse should tell a primigravida that the definitive sign indicating that labor has begun is what? A. Progressive uterine contractions with cervical change. B. Lightening. C. Rupture of membranes. D.Passage of the mucus plug (operculum).

Regular, progressive uterine contractions that increase in intensity and frequency are the definitive sign of true labor along with cervical change. Lightening is a premonitory sign indicating that the onset of labor is getting closer. Rupture of membranes usually occurs during labor itself. Passage of the mucus plug is a premonitory sign indicating that the onset of labor is getting closer (A)

*Challenge* A physical care measure for a laboring woman that has been identified as unlikely to be beneficial and may even be harmful is: A. Allowing the laboring woman to drink fluids and eat light solids as tolerated. B. Administering a Fleet enema at admission. C. Ambulating periodically throughout labor as tolerated. D. Using a whirlpool bath once active labor is established.

Research has indicated that enemas are not needed during labor; according to research findings, choices A, C, and D have all been found to be beneficial and safe during pregnancy. B. Administering a Fleet enema at admission.

A client who is at 30 weeks gestation reports a brownish cervical discharge. What questions does the nurse ask first in order to identify the cause of the discharge? A. "Are you experiencing abdominal pain?" B. "Are you experiencing strong uterine contractions?" C. "Did you have sexual intercourse in the last two days?" D. "Did you take any other medications than those prescribed?"

Sexual intercourse during this period of the pregnancy may cause cervical trauma and result in cervical discharge. Therefore, the nurse should first ask whether the client had intercourse in the last two days. Abdominal pain and strong uterine contractions are indicative of labor, but the client is unlikely to go into labor at this time. Medications should not generally cause production of brownish cervical discharge, so this is not a priority question at this time. (C)

What are signs of potential complications of labor? Select all that apply. A. Contractions lasting 90 seconds or longer B. Urine-smelling vaginal discharge C. Irregular fetal heart rate (FHR); suspected fetal arrhythmias D. More than five contractions in a 5-minute period E. Relaxation between contractions lasting shorter than 30 seconds F. Intrauterine pressure of ≥80 mm Hg or resting tone of ≥20 mm Hg (both determined by internal monitoring with intrauterine pressure catheter [IUPC])

Signs of potential complications of labor include contractions lasting 90 seconds or longer; irregular FHR and suspected fetal arrhythmias; relaxation between contractions lasting shorter than 30 seconds; and intrauterine pressure of ≥80 mm Hg or resting tone of ≥20 mm Hg (both determined by internal monitoring with IUPC). Foul-smelling (not urine-smelling) vaginal discharge, and more than five contractions in a 10-minute (not 5-minute) period (contractions occurring more frequently than every 2 minutes) are also signs of potential complications of labor. (A, C, E, F)

On completion of a vaginal examination on a laboring woman, the nurse records: 50%, 6 cm, -1. What is a correct interpretation of the data? A. The fetal presenting part is 1 cm above the ischial spines. B. Effacement is 4 cm from completion. C. Dilation is 50% completed. D. The fetus has achieved passage through the ischial spines.

Station of -1 indicates that the fetal presenting part is above the ischial spines and has not yet passed through the pelvic inlet. Progress of effacement is referred to by percentages, with 100% indicating full effacement and dilation by centimeters, with 10 cm indicating full dilation. Progress of effacement is referred to by percentages, with 100% indicating full effacement and dilation by centimeters, with 10 cm indicating full dilation. Passage through the ischial spines with internal rotation would be indicated by a plus station such as +1. (A)

The nurse is providing care for a client with twins during labor. The nurse instructs the client to avoid lying flat on the back. Which condition does the nurse aim to prevent in the client during labor? A. Valsalva maneuver B. Supine hypotension C. Respiratory alkalosis D. Painful uterine contractions

Supine hypotension may be seen in the pregnant client when the ascending vena cava and descending aorta are compressed. The client is more at risk for hypotension in the case of multifetal pregnancy due to a drop in hydrostatic pressure when lying supine. Valsalva maneuver refers to the client holding her breath and tightening the abdominal muscles to help with pushing. Respiratory alkalosis may occur in a pregnant client due to hyperventilation and not by lying flat on the back. Epidural analgesia is often prescribed to decrease pain experienced during uterine contractions. (B)

While monitoring the fetal heart rate (FHR) of a client, the nurse notes tachycardia. What is a probable cause for this condition? A. Early signs of fetal distress B. Maternal hypothermia C. Maternal hypoglycemia D. Atrioventricular dissociation

Tachycardia is a baseline FHR greater than 160 beats/minute that lasts for 10 minutes or longer. It may be considered an early sign of fetal distress or even fetal hypoxemia, especially when associated with late decelerations and minimal or absent variability. It can result from maternal or fetal infection. Bradycardia is a baseline FHR less than 110 beats/minute that lasts for 10 minutes or longer. Maternal hypothermia or maternal hypoglycemia may cause bradycardia. Bradycardia, not tachycardia, is often caused by some type of fetal cardiac problem. These may include structural defects involving the conduction system, as in atrioventricular dissociation. (A)

*Challenge* The primary health care provider prescribes terbutaline (Brethine) for a pregnant client. As the nurse reviews the client's medical record, what would be the rationale for this prescription? A. Blood volume is elevated. B. Hemoglobin is decreased. C. Blood pressure is reduced. D. Contractions are increased.

Terbutaline (Brethine) is administered to the client who has premature labor. It slows down the contractions. Terbutaline (Brethine) has no effect on blood volume, blood pressure, and hemoglobin. Blood volume is elevated by infusing aggressive intravenous infusion or from sodium and water retention. Maternal hypotension, as evidenced by reduced blood pressure, is relieved by elevating the legs during labor. Clients with decreased hemoglobin are treated with iron supplements, not terbutaline (Brethine). (D)

Nurses can help their clients by keeping them informed about the distinctive stages of labor. What description of the phases of the first stage of labor is accurate? A. Latent: mild, regular contractions; no dilation; bloody show; duration of 2 to 4 hours B. Active: moderate, regular contractions; 4 to 7 cm dilation; duration of 3 to 6 hours C. Lull: no contractions; dilation stable; duration of 20 to 60 minutes D. Transition: very strong but irregular contractions; 8 to 10 cm dilation; duration of 1 to 2 hours E. Full cervical dilation marks the end of the first stage of labor

The active stage is characterized by moderate, regular contractions; 4 to 7 cm dilation; and a duration of 3 to 6 hours. The latent phase is characterized by mild to moderate, irregular contractions; dilation up to 3 cm; brownish to pale pink mucus; and a duration of 6 to 8 hours. The transition phase is characterized by strong to very strong, regular contractions; 8 to 10 cm dilation; and a duration of 20 to 40 minutes. Full cervical dilation marks the end of the first stage of labor. No official "lull" phase exists in the first stage. (B)

The nurse is performing the pelvic examination of a client during the prenatal visit. Which pelvic type is heart shaped and least favorable for a vaginal birth? A. Gynecoid B. Android C. Anthropoid D. Platypelloid

The android pelvis is heart shaped and angulated. The sidewalls are convergent; the sacrum is slightly curved and the terminal portion is often beaked. The subpubic arch is narrow, often resulting in cesarean births or difficult vaginal forceps birth. It is least favorable for vaginal birth. The gynecoid pelvis is slightly ovoid or transversely rounded. The side walls are straight, sacrum deep and curved. The subpubic arch is wide, thus enabling spontaneous vaginal births. The anthropoid pelvis is oval and wider anteroposteriorly. The sidewalls are straight, sacrum slightly curved. The subpubic arch is narrow and may result in forceps vaginal birth. The platypelloid pelvis is flattened anteroposteriorly and wide transversely. The side walls are straight, the sacrum slightly curved, and the subpubic arch is wide, resulting in spontaneous vaginal birth. (B)

The nurse palpates the fontanels and sutures to determine the fetal presentation. What is the feature of the anterior fontanel? A. It is diamond-shaped in appearance. B. It measures about 1 cm by 2 cm. C. It closes after 6 to 8 weeks of birth. D. It lies near the occipital bone.

The anterior fontanel is diamond-shaped and measures about 3 cm by 2 cm. It closes by 18 months after birth. It lies at the junction of the sagittal, coronal, and frontal sutures. The posterior fontanel is triangular in shape and measures about 1 cm by 2 cm. It closes after 6 to 8 weeks of birth. It lies at the junction of the sutures of the two parietal bones and the occipital bone. (A)

When using intermittent auscultation (IA) to assess uterine activity, nurses should be aware that: A. The examiner's hand should be placed over the fundus before, during, and after contractions. B. The frequency and duration of contractions are measured in seconds for consistency. C. Contraction intensity is given a judgment number of 1 to 7 by the nurse and client together. D. The resting tone between contractions is described as either placid or turbulent.

The assessment is done by palpation; duration, frequency, intensity, and resting tone must be assessed. The duration of contractions is measured in seconds; the frequency is measured in minutes. The intensity of contractions usually is described as mild, moderate, or strong. The resting tone usually is characterized as soft or relaxed. (A)

When using intermittent auscultation to assess uterine activity, nurses should be aware of what? A. The examiner's hand should be placed on the fundus before, during, and after contractions B. The frequency and duration of contractions are measured in seconds for consistency C. Contraction intensity is given a judgment number of 1 to 7 by the nurse and client together D. The resting tone between contractions is described as either placid or turbulent

The assessment is done by palpation; duration, frequency, intensity, and resting tone must be assessed. The duration of contractions is measured in seconds; the frequency is measured in minutes. The intensity of contractions usually is described as mild, moderate, or strong. The resting tone usually is characterized as soft or relaxed. (A)

The nurse is monitoring the fetal heart rate (FHR) of a client who is in labor at full term. What measure does the nurse take to obtain the most accurate baseline fetal heart rate? A. Record or monitor a 10-minute segment of tracing. B. Include periods of marked variability in the segment. C. Include episodic changes in the segment of tracing. D. Obtain at least 5 minutes of interpretable data in the segment.

The baseline fetal heart rate is the average rate during a 10-minute segment, and that is why the nurse must obtain a 10-minute segment of tracing to determine the baseline FHR. In order to determine a baseline heart rate, the 10-minute segment must not include periods of marked variability or periodic or episodic changes. The nurse must ensure there are at least 2 minutes of interpretable baseline data in a 10-minute segment of tracing. (A)

What should be included in the birth plan? Select all that apply. Presence of birth companions and the role each will play Presence of other persons such as other clients and students Cultural and religious requirements related to the care of the mother, newborn, and placenta Labor activities such as preferred positions for labor and birth, ambulation, birth balls, showers and whirlpool baths, oral food and fluid intake Labor and birth medical interventions such as pharmacologic pain relief measures, intravenous therapy, electronic monitoring, induction or augmentation measures, and episiotomy Care and handling of the newborn immediately after birth such as immediate skin-to-skin contact, cutting of the cord, eye care, and breastfeeding

The birth plan should include the presence of birth companions and the role each will play; the presence of other persons such as students or male attendants (not other clients); cultural and religious requirements related to the care of the mother, newborn, and placenta; labor activities such as preferred positions for labor and birth, ambulation, birth balls, showers and whirlpool baths, and oral food and fluid intake; labor and birth medical interventions such as pharmacologic pain relief measures, intravenous therapy, electronic monitoring, induction or augmentation measures, and episiotomy; and care and handling of the newborn immediately after birth, such as immediate skin-to-skin contact, cutting of the cord, eye care, and breastfeeding. (A, C, D, E, F)

Which of the following would be included in a birth plan for an expectant mother? Select all that apply. A. Tracking of the onset of progressive, regular contractions B. The presence of birth companions such as the partner C. Preferred position for labor and for birth D. Medical interventions E. Discussion of any fears about labor

The birth plan should include the woman's or couple's preferences related to presence of birth companions such as the partner, preferred position for labor and birth, and medical interventions. The birth plan does not include the tracking of the onset of progressive, regular contractions or discussion of any fears about labor. (B, C, E)

The nurse is caring for a client who had a normal vaginal birth. The client is concerned about the shape of the infant's head. What does the nurse tell the client? Select all that apply. A. The bones of the skull continue to grow after birth. B. The shape of the head undergoes molding during labor. C. The head assumes its normal shape within a month. D. The skull bones of an infant are generally firmly united. E. The sutures and fontanels make the skull flexible.

The bones of the skull continue to grow for some time after birth to accommodate the infant's brain. During labor, the shape of the head gets molded as the bones undergo a slight overlapping. The sutures and fontanels are membranous structures that unite the skull bones and make the skull flexible. Molding can be extensive, but the heads of most newborns assume their normal shape within 3 days after birth. The skull bones are held together by sutures and fontanels and are not firmly united in an infant. (A, B, E)

The nurse is providing care for a client in the first stage of labor. The client's prenatal documentation indicates that the client has scarring on her cervix due to a past STI. What complication might the nurse predict in the client during labor? A. Ferguson reflex B. Slow fetal descent C. Supine hypotension D. Slow cervical dilation

The cervical dilation is slowed if a previous vaginal infection has caused scarring of the cervix. This is because the dilation occurs by the drawing upward of the musculofibrous components of the cervix. Ferguson reflex refers to the maternal urge to bear down when the stretch receptors in the posterior vagina release endogenous oxytocin. The administration of epidural analgesia may slow the rate of fetal descent. Supine hypotension occurs due to a drop in hydrostatic pressure. (D)

Which statement by the student nurse about the cervix indicates effective learning? A. "The cervix allows fetal descent into the vagina." B. "The cervix is the external opening of the vagina." C. "The cervix allows the fetus to rotate anteriorly." D. "The cervix distends and accommodates the intrauterine contents."

The cervix is a soft tissue that effaces (thins) and dilates (opens) to allow fetal descent into the vagina. The introitus is the external opening of the vagina. The pelvic floor muscles allow the fetus to rotate anteriorly and help it to pass through the birth canal. The lower uterine segment distends and accommodates the intrauterine contents when the wall of the upper segment thickens. (A)

The nurse is assisting a client in labor. What neurologic changes does the nurse expect in the laboring client? A. Decreased pain threshold. B. Amnesia and sedation. C. Increased perception of pain. D. Client is elated between contractions.

The client experiences amnesia between contractions in the second stage of labor. Endogenous endorphins produced by the body cause sedation. This also raises the pain threshold. Pressure of the presenting part causes physiologic anesthesia of the perineal tissues. This decreases the perception of pain. At the start of labor, the client may be euphoric. Euphoria first gives way to increased seriousness. Second, it gives way to amnesia between contractions. Finally, it leads to elation or fatigue after giving birth. (B)

A pregnant woman who is in the first stage of labor has a body weight of 73 kg. While reviewing the medical reports, the nurse finds that the client's prepregnant body weight was 53 kg. What does the nurse interpret about the client? A. The client has an increased risk of prolonged labor. B. The client has an increased risk of caesarean delivery. C. The client is likely to require oxytocin to induce labor. D. The client is likely to have stronger uterine contractions during labor.

The client has gained 20 kg over the 9 months of pregnancy. Weight gain of more than 16 kg during pregnancy indicates that the client is at high risk of cephalopelvic disproportion, and may require a caesarean delivery. Prolonged labor may be caused by breech presentation or occiput posterior position of the fetus. Inadequate uterine contractions or weak uterine contractions indicate a need for oxytocin administration during delivery. The strength of uterine contractions is not dependent on weight gain during pregnancy. (B)

The nurse encourages a client to experiment with various positions during labor. What is the rationale behind this instruction? A. To enhance gas exchange in the fetus B. To assist downward movement of the fetus C. To reduce anxiety and fear in the client D. To prevent cervical and vaginal lacerations

The client is encouraged to experiment with various positions during the process of labor to help the labor progress and to remain comfortable. Certain maternal positions help improve placental sufficiency more than others. These help hasten the process of vaginal delivery.To enhance gas exchange in the fetus, the client is asked to take deep, cleansing breaths. Experimenting with various positions does not aid in effective breathing. Anxiety and fear are common during the process of labor, but this intervention may not reduce anxiety in the client. The client is made to feel comfortable by minimizing distractions during labor. Changing of positions may not help in preventing lacerations sustained during the childbirth process. (B)

The nurse is using auscultation to determine the fetal heart rate (FHR) during the first stage of labor. What measures can the nurse use to reassure the mother if it takes considerable time to locate and count the heartbeats? A. Ask the health care provider to locate the heartbeat. B. Let the mother know the sounds are muffled. C. Use internal monitoring to locate the heartbeat. D. Allow the mother to listen to the heartbeat.

The client may become anxious if the nurse takes considerable time to locate and count the fetal heartbeats. The nurse can reassure the mother by allowing the mother to listen to the heartbeat after it is located. The nurse may seek assistance if necessary to identify the fetal heartbeat; however, the nurse is usually capable of locating the heartbeat with patience, and escalating the intervention to a health care provider can sometimes increase anxiety in the client. The nurse must let the mother know that it takes time to identify the spot with the loudest and clearest heartbeats that can be counted. The nurse need not tell the mother that the sounds are muffled. The nurse must use an ultrasound to locate the heartbeat during the first stage of labor. Internal monitoring is possible only when the cervix is dilated sufficiently and the membranes are ruptured. (D)

The nurse is providing care for a client in labor. What does the nurse instruct the client in the second stage of labor? A. "Point your toes, to prevent pain." B. "Avoid fluids until the infant is delivered." C. "Lie still and avoid movement to prevent fatigue." D. "Avoid holding your breath or tightening the abdominal muscles."

The client may hold her breath and tighten the abdominal muscles for pushing during the second stage of labor. This activity is known as the Valsalva maneuver. The activity increases intrathoracic pressure, reducing venous return while increasing venous pressure. Therefore, the nurse instructs the client to avoid the Valsalva maneuver. The client should not point her toes, because it may cause leg cramps. The client should not avoid fluids if thirsty, because it may cause dehydration. The nurse should instruct the client to change positions every few minutes in order to facilitate delivery during the second stage of labor. (D)

What instructions does the nurse give to reduce the risk of urinary retention in a client during labor? A. Ask the client to void in a lateral position. B. Ask the client to increase her fluid intake. C. Ask the client to empty her bladder every 4 hours. D. Assist the client with techniques to help stimulate voiding.

The client may not feel the urge to void during labor. The nurse should palpate the bladder for distention and ask the client to void spontaneously by using voiding techniques like running water. The nurse should ask the client to void in an upright position rather than in a lateral position. The client should not increase her fluid intake, because it may not help in voiding. The nurse should ask the client to empty her bladder every two hours. (D)

When assessing a client for the possibility of a vaginal birth, what must the nurse keep in mind about the coccyx of the bony pelvis? A. It is the part above the brim of the bony pelvis. B. It is movable in the latter part of the pregnancy. C. It has three planes, the inlet, midpelvis, and outlet. D. It is ovoid and bound by pubic arch anteriorly.

The coccyx is movable in the latter part of the pregnancy, unless it has been broken and fused to the sacrum during healing. The bony pelvis is separated by the brim into the false and the true pelves. The false pelvis is the part above the brim and plays no part in childbearing. The true pelvis is involved in birth and is divided into three planes: inlet, midpelvis, and outlet. The pelvic outlet is the lower border of the true pelvis. Viewed from below it is the ovoid. It is shaped somewhat like a diamond and bound by the pubic arch anteriorly, the ischial tuberosities laterally, and the tip of the coccyx posteriorly. (B)

What are the differences between true and false labor? A. There are no differences between true and false labor. B. The differences between true labor and false labor include: true labor—contractions occur quickly, vaginal exam reveals appearance of bloody show, and the head of the fetus is visible in the pelvis. False labor—contractions are slow, vaginal exam has some bloody show, and the fetus is not visible in the pelvis. C. In true labor, contractions occur irregularly; they can be felt in the back or abdomen; the cervix may be soft, but there is no effacement or dilation; and the fetus is not engaged in the pelvis. In false labor, contractions occur regularly, become stronger, last longer, and occur closer together; they are usually felt in the lower back; the vaginal exam shows softening, effacement, and dilation by appearance of bloody show; and the fetus becomes engaged in the pelvis. D. In true labor, contractions occur regularly, become stronger, last longer, and occur closer together; they are usually felt in the lower back; the vaginal exam shows softening, effacement, and dilation by appearance of bloody show; and the fetus becomes engaged in the pelvis. In false labor, contractions occur irregularly; they can be felt in the back or abdomen; the cervix may be soft, but there is no effacement or dilation; and the fetus is not engaged in the pelvis.

The differences between true and false labor include the following: In true labor, contractions occur regularly, become stronger, last longer, and occur closer together; they are usually felt in the lower back; the vaginal exam shows softening, effacement, and dilation by the appearance of bloody show; and the fetus becomes engaged in the pelvis. In false labor, contractions occur irregularly; they can be felt in the back or abdomen; the cervix may be soft, but there is no effacement or dilation; and the fetus is not engaged in the pelvis. There are definite differences between true and false labor. (D)

What action does the nurse take when using a tocotransducer on a client during preterm labor? A. Assist the client to lie in a supine position. B. Palpate to assess the contraction frequency. C. Secure the device just above the umbilicus. D. Rely on the device to identify uterine activity.

The fetal monitor of the tocotransducer is designed to assess uterine activity (UA) in full-term pregnancy. Because it is not always sensitive enough to detect preterm UA, the nurse may use palpation as an additional way of assessing contraction frequency and validating monitor tracing. The device confines the client to a bed or a chair. The client is assisted into a semi-Fowler or lateral position for comfort. If a pregnant patient is placed in a supine position, it can cause hypotension. The device is placed over the fundus above the umbilicus for a patient in term labor. However, the fundus may be below the umbilicus in a client with preterm labor. The nurse must rely on the client and use palpation to identify UA, rather than depend on the device. (B)

A postpartum client is worried because her newborn's head has an abnormal shape instead of being round. The delivery documentation indicates that the newborn had molding upon delivery. What is nurse's best response? A. "The infant will look better after more hair grows." B. "The infant's skull needs to be massaged after a month." C. "The infant's head will assume a normal shape in 3 days." D. "Some infants have an oddly shaped head, which is alright."

The fetal skull bones are not completely fused and there may be a slight overlapping of the bones during the labor process. This causes molding of the fetal head. The molding is not permanent and the infant's head assumes a normal shape within 3 days of birth. Telling the client that the infant will look better after hair growth will not help alleviate the client's anxiety about the fetal head. The infant's head will assume a normal shape within 3 days, and the client need not wait for a month to massage the infant's head. Some infants may have an oddly shaped head, but in this case, the molding has occurred due to the labor. (C.)

A client has just vaginally delivered a 6-lb baby girl and the placenta. What does the fourth stage of labor entail? Select all that apply. A. It is a crucial time for mother and newborn. B. The fourth stage of labor is delivery of the fetus. C. The fourth stage of labor includes delivery of the placenta. D. The fourth stage of labor includes the first 1 to 2 hours after birth. E. During this time, maternal organs undergo their initial readjustment to the nonpregnant state, and the functions of body systems begin to stabilize. F. Mother and baby are not only recovering from the physical process of birth, but also becoming acquainted with each other and additional family members.

The fourth stage of labor is a crucial time for the mother and the newborn; it includes the first 1 to 2 hours after birth. During this time maternal organs undergo their initial readjustment to the nonpregnant state and the functions of body systems begin to stabilize. The mother and baby are not only recovering from the physical process of birth, but are also becoming acquainted with each other and additional family members. The second (not fourth) stage of labor is delivery of the fetus. The third (not fourth) stage of labor includes delivery of the placenta. (A, D, E, F)

Which pelvic shape is the most classic female pelvis shape and most conducive to vaginal labor and birth? A. Android B. Gynecoid C. Platypelloid D. Anthropoid

The gynecoid pelvis is round and cylinder shaped, with a wide pubic arch. Prognosis for vaginal birth is good. Only 23% of women have an android-shaped pelvis, which has a poor prognosis for vaginal birth. The platypelloid pelvis is flat, wide, short, and oval. The anthropoid pelvis is a long, narrow oval with a narrow pubic arch. It is more favorable than the android or platypelloid pelvic shape. (B)

If a woman complains of back labor pain, the nurse might best suggest that she: A. Lie on her back for a while with her knees bent. B. Do less walking around. C. Take some deep, cleansing breaths. D. Lean over a birth ball with her knees on the floor.

The hands-and-knees position, with or without the aid of a birth ball, should help with the back pain. The supine position should be discouraged. Walking generally is encouraged. Deep cleansing breaths will assist with any labor pain; however, it is very important that this woman's position is changed so that she is not on her back. (D)

**Challenge** Which is one of the best positions in which to place the mother in case of complications during delivery? A. Prone position B. Maternal position C. Lithotomy position D. Side-lying position

The lithotomy position makes dealing with some complications that arise more convenient for the nurse-midwife or physician. To place the woman in this position, bring her buttocks to the edge of the bed or table and place her legs in stirrups. Prone and side-lying positions are not best for complications during delivery. The maternal position for birth in a birthing room varies from a lithotomy position, with the woman's feet in stirrups or resting on footrests, or with her legs held and supported by the nurse or support person, to one in which her feet rest on footrests while she holds on to a squat bar, to a side-lying position with the woman's upper leg supported by the coach, nurse, or squat bar. (C)

Which is an abnormal finding in a fetus during labor? A. The fetal heart rate is 190 beats/minute at term. B. The fetal head is in a synclitic position. C. The fetal oxygen pressure decreases. D. The fetal circulation is decreased.

The normal range of fetal heart rate is 110 to 160 beats/minute at term. Therefore, 190 beats/minute is an abnormal finding in the fetus. The fetal head is usually in a synclitic position, which indicates that the head is parallel to the anteroposterior plane of the pelvis. The oxygen pressure decreases as the fetal lung fluid is cleared from the air passage during the birth process. This aids in immediate respiration after birth. The fetal circulation tends to decrease during labor because of uterine contractions. (A)

Which fetal and maternal physiologic conditions does the nurse assess during the first stage of labor? Select all that apply. A. Fetal heart rate B. Fetal circulation C. Maternal heart rate D. Maternal Valsalva maneuver E. Maternal supine hypotension

The nurse assesses fetal heart rate to evaluate oxygen demands. Maternal position, uterine contractions, blood pressure, and umbilical cord blood flow may affect fetal circulation. There is a drop in the maternal heart rate during labor and the nurse should monitor it to be alert to any complications. The Valsalva maneuver may be seen in the patient during the second stage of labor. The client may hold her breath and tighten the abdominal muscles for pushing. Supine hypotension may be seen in the client during labor if the uterus is large or the client is obese or hypovolemic. (A, B, C)

The nurse is palpating for uterine activity during intermittent auscultation (IA) for a client in labor. What interventions does the nurse perform? Select all that apply. A. Place a scope over the fundus to assess resting tone. B. Document the uterine resting tone as either hard or soft. C. Place a hand over the fundus during the contractions. D. Measure the uterine contraction duration in seconds. E. Measure the uterine contraction frequency in minutes.

The nurse assesses the resting tone, or relaxation between contractions, and documents the uterine resting tone as hard or soft. The duration of a uterine contraction, from the beginning to the end of the contraction, is measured in seconds. The frequency of contractions is measured in minutes from the beginning of one contraction to the beginning of the next contraction. The nurse assesses the resting tone by placing a hand, not a scope, over the fundus after the contraction is over. The nurse places fingertips or a hand over the fundus before, during, and after the contraction. This helps the nurse assess the intensity, duration, and frequency of the contractions. (B, C, D, E)

The nurse is assisting a pregnant client in labor. What instructions should the nurse give to the client to promote comfort? Select all that apply. A. "You should cough frequently." B. "Breathe with your mouth open." C. "Lie down in the lateral position." D. "Lie in the supine position in bed." E. "Lie in the semi-Fowler's position."

The nurse helps the pregnant client during labor. This includes teaching the client relaxation techniques. The nurse teaches the client to keep the mouth open during exhalation to allow air to easily leave the lungs. Placing the client in a semi-Fowler's or lateral position is helpful during labor. Therefore the nurse should instruct the client to maintain the lateral or semi-Fowler's position with a lateral tilt. Asking the client to cough frequently would increase the intraabdominal pressure of the client and would make the client uncomfortable. Having the client lie down in a supine position during labor may cause orthostatic hypotension. Therefore the nurse should instruct the client to lie down in a position other than supine. (B, C, E)

Which technique can the nurse use to stimulate the fetus and accelerate the fetal heart rate (FHR)? A. Discontinue the administration of oxytocin. B. Place the client in the knee chest position. C. Administer medications for tocolytic therapy. D. Use a halogen light over the maternal abdomen.

The nurse may place a specialized halogen light source on the maternal abdomen to stimulate the fetus and accelerate the FHR by at least 15 beats/minute for at least 15 seconds. Oxytocin infusion is discontinued to reduce uterine contractions. The client is placed in the knee chest position to relieve compression on the umbilical cord. Tocolytic therapy involves the administration of drugs that inhibit uterine contractions. They are used to relax the uterus. (D)

Which nursing intervention is useful for a client in labor to reduce intrathoracic pressure and prevent fetal hypoxia? A. Encourage the client to take deep, cleansing breaths and relax. B. Encourage the client to exhale, holding her breath for short periods. C. Encourage the client to position herself comfortably. D. Encourage the client to push when she feels the urge to bear down.

The nurse should encourage the client to exhale after holding her breath for short periods. This prevents a sudden increase in the client's intrathoracic pressure and prevents fetal hypoxia. Encouraging the client to take deep, cleansing breaths and relaxing between contractions helps reduce fatigue and increase the effectiveness of pushing efforts during labor. Encouraging the client to maintain a comfortable position during labor facilitates the labor process. Encouraging the client to push when she perceives the urge to bear down helps aid in the descent and rotation of the fetus. (B)

During the vaginal examination of a client in labor, the nurse identifies the presenting part as the scapula. Which fetal presentation does the nurse recognize? A. Cephalic B. Frank breech C. Complete breech D. Shoulder

The presenting part can be defined as that part of the fetus that lies closest to the internal os of the cervix. In the shoulder presentation, the presenting part is the scapula. In a cephalic presentation, the presenting part is usually the occiput. In a breech presentation, the presenting part is the sacrum. The sacrum is the presenting part in a frank breech presentation. The sacrum and feet are the presenting parts in a complete breech presentation. (D)

With regard to primary and secondary powers, the maternity nurse should understand that: A. Primary powers are responsible for effacement and dilation of the cervix. B. Effacement generally is well ahead of dilation in women giving birth for the first time; they are more together in subsequent pregnancies. C. Scarring of the cervix caused by a previous infection or surgery may make the delivery a bit more painful, but it should not slow or inhibit dilation. D. Pushing in the second stage of labor is more effective if the woman can breathe deeply and control some of her involuntary needs to push, as the nurse directs.

The primary powers are responsible for dilation and effacement; secondary powers are concerned with expulsion of the fetus. Effacement is generally well ahead of dilation in first-timers; the two are more concurrent in subsequent pregnancies. Scarring of the cervix may slow dilation. Pushing is more effective and less fatiguing when the woman begins to push only after she has the urge to do so. (A)

What are the functions of the primary powers in the labor process? Select all that apply. A. Dilation of the cervix B. Descent of the fetus C. Effacement of the cervix D. Raise the pain threshold E. Expulsion of the infant from the uterus

The primary powers efface and dilate the cervix at the onset of labor. Dilation refers to the enlargement of the cervical opening. The diameter of the cervix increases from less than 1 cm to 10 cm. The primary powers increase UCs that help in the descent of the fetus. Effacement refers to the shortening and thinning of the cervix during the first stage of labor. The primary powers along with the secondary powers are used in the expulsion of the infant from the uterus. Endogenous endorphins that are naturally produced by the body raise the pain threshold in the patient. (A, B, C, E)

The nurse knows that the second stage of labor, the descent phase, has begun when: A. The amniotic membranes rupture. B. The cervix cannot be felt during a vaginal examination. C. The woman experiences a strong urge to bear down. D. The presenting part is below the ischial spines.

The second stage of labor begins with full cervical dilation. During the active pushing phase of the second stage of labor, the woman may experience an increase in the urge to bear down. Rupture of membranes has no significance in determining the stage of labor. Many women may have an urge to bear down when the presenting part is below the level of the ischial spines. This can occur during the first stage of labor, as early as at 5 cm dilation. (B)

The nursing instructor asks a student about the different stages of labor. Which statement by the student indicates effective learning? A. "There is no abnormal bleeding in the first stage of labor." B. "The placenta is delivered in the fourth stage of labor after the birth." C. "The full effacement and dilation of the cervix indicates the beginning of the second stage." D. "The second stage lasts from full dilation of the cervix to the birth of the fetus."

The second stage of labor is composed of two phases: the latent (passive fetal descent) phase and the active pushing phase. In the latent phase, the fetus continues to descend passively through the birth canal, rotating in an anterior position due to the uterine contractions. In the active pushing phase, the fetus presses on the stretch receptors of the pelvic floor. Abnormal bleeding may sometimes occur in the first stage of labor which needs prompt attention by the primary health care provider. The placenta separates in the third stage of the labor after the birth of the fetus. The full effacement and dilation of the cervix ends at the first stage of the labor. (D)

When is the best time to determine the station of the presenting part in a pregnant client? A. When the labor begins B. A week before the labor C. During the fourth stage of labor D. At the end of the third stage of labor

The station is the relationship of the presenting fetal part to an imaginary line drawn between the maternal ischial spines. The best time to determine the station is when the labor begins, because it helps to accurately determine the rate of fetal descent. Birth is imminent when the presenting part is at +4 cm to +5 cm below the spine. A week before the labor is too early to determine the station, because fetal descent has usually not begun. The delivery of the placenta occurs in the fourth stage of labor. Therefore, the birth process is already complete by this stage. The third stage involves the birth of the infant and ends with the expulsion of the placenta. Therefore, it is ineffective to determine the station at that point. (A)

Concerning the third stage of labor, nurses should be aware that: A. The placenta eventually detaches itself from a flaccid uterus. B. The duration of the third stage may be as short as 3 to 5 minutes. C. It is important that the dark, roughened maternal surface of the placenta appear before the shiny fetal surface. D. The major risk for women during the third stage is a rapid heart rate.

The third stage of labor lasts from birth of the fetus until the placenta is delivered. The duration may be as short as 3 to 5 minutes, although up to 1 hour is considered within normal limits. The placenta cannot detach itself from a flaccid (relaxed) uterus. Which surface of the placenta comes out first is not clinically important. The major risk for women during the third stage of labor is postpartum hemorrhage; the risk of hemorrhage increases as the length of the third stage increases. (B)

What are the different parts of the true pelvis? Select all that apply. A. Brim B. Outlet C. Android D. Midpelvis E. Anthropoid

The true pelvis is involved in the birth process and is divided into three planes: the inlet, or brim; the midpelvis, or cavity; and the outlet. The brim or the pelvic inlet is the upper border of the true pelvis. It is made up of the upper margins of the pubic bone anteriorly and the iliopectineal lines along the innominate bones laterally. Posteriorly, it has the anterior upper margin of the sacrum and the sacral promontory. The pelvic outlet is the lower border of the true pelvis. It is ovoid and diamond shaped. It has the pubic arch anteriorly, the ischial tuberosities laterally, and the tip of the coccyx posteriorly. The midpelvis, or the pelvic cavity, is a curved passage. It has a short anterior wall and a longer concave posterior wall. It has the posterior aspect of the symphysis pubis, the ischium, a portion of the ilium, the sacrum, and the coccyx. Android and anthropoid are the different basic types of the pelvis. (A, B, D)

Evidence-based care practices designed to support normal labor and birth recommend which practice during the immediate newborn period? A. The healthy newborn should be taken to the nursery for a complete assessment. B. After drying, the infant should be given to the mother wrapped in a receiving blanket. C. Skin-to-skin contact of mother and baby should be encouraged. D. The father or support person should be encouraged to hold the infant while awaiting delivery of the placenta.

The unwrapped infant should be placed on the woman's bare chest or abdomen, then covered with a warm blanket. Skin-to-skin contact keeps the newborn warm, prevents neonatal infection, enhances physiologic adjustment to extrauterine life, and fosters early breastfeeding. Although complete assessment in the nursery is the practice in many facilities, it is neither evidence-based nor supportive of family-centered care. Handing the mother the blanket-wrapped baby is a common practice and more family friendly than separating mother and baby; however, ideally the baby should be placed on the mother skin to skin. The father or support person is likely also anxious to hold and admire the newborn. This can happen after the infant has been placed skin to skin with the mother and breastfeeding has been initiated. (C)

A nurse caring for a woman in labor understands that increased variability of the fetal heart rate might be caused by: (Hint: Depressants vs. Stimulants) A. Narcotics. B. Barbiturates. C. Methamphetamines. D. Tranquilizers.

The use of illicit drugs such as cocaine or methamphetamines might cause increased variability. Maternal ingestion of narcotics and tranquilizer use may be the causes of decreased variability. The use of barbiturates may also result in a significant decrease in variability as these drugs are known to cross the placental barrier. (C)

A nurse caring for a woman in labor understands that increased variability of the fetal heart rate might be caused by what? A. Narcotics B. Barbiturates C. Methamphetamines D. Tranquilizers

The use of illicit drugs such as cocaine or methamphetamines might cause increased variability. Maternal ingestion of narcotics may be the cause of decreased variability. The use of barbiturates may also result in a significant decrease in variability, because these are known to cross the placental barrier. Tranquilizer use is a possible cause of decreased variability in the fetal heart rate. (C)

The nurse is assisting a client during the second phase of labor. Which behavior from the client signifies the active pushing stage of the second phase of labor? A. The client may scream or swear. B. The client feels fatigued and sleepy. C. The client is inattentive to the nurse's instructions. D. The client is quiet, and is anxious about the progression of labor.

There are two phases during the second stage of labor, the latent and the active. The active stage is the pushing stage of labor, in which the client experiences severe pain and tries to push the fetus with all her effort. The client is inattentive to the nurse and directs all her concentration on childbirth. During the active phase of vocalization, the client may scream or swear, because the pain is severe.The client feels fatigued and sleepy during the latent phase, not during the active phase. During the latent phase, the client remains quiet and is concerned with the progress of the labor. (C)

You are evaluating the fetal monitor tracing of your client, who is in active labor. Suddenly you see the fetal heart rate (FHR) drop from its baseline of 125 down to 80. You reposition the mother, provide oxygen, increase IV fluid, and perform a vaginal exam. The cervix has not changed. Five minutes have passed, and the FHR remains in the 80s. What additional nursing measures should you take? A. Call for help. B. Insert a Foley catheter. C. Start oxytocin (Pitocin). D. Notify the primary health care provider immediately.

To relieve an FHR deceleration the nurse can reposition the mother, increase IV fluid, and provide oxygen. Also, if oxytocin is being infused, it should be discontinued. If the FHR does not resolve, the primary health care provider should be notified immediately. Although it is always a good idea to have extra help during any unanticipated obstetric event, calling for help is not the most important nursing measure at this time. If the FHR were to continue in an abnormal or nonreassuring pattern, a cesarean section might be warranted. This would require the insertion of a Foley catheter; however, the physician must make that determination. Oxytocin may put additional stress on the fetus. (D)

You are evaluating the fetal monitor tracing of your client, who is in active labor. Suddenly you see the fetal heart rate (FHR) drop from its baseline of 125 down to 80. You reposition the mother, provide oxygen, increase IV fluid, and perform a vaginal exam. The cervix has not changed. Five minutes have passed, and the FHR remains in the 80s. What additional nursing measures should you take? A. Call for help. B. Insert a Foley catheter. C. Start oxytocin (Pitocin). D. Notify the primary health care provider immediately.

To relieve an FHR deceleration the nurse can reposition the mother, increase IV fluid, and provide oxygen. In addition, if oxytocin is infusing, it should be discontinued. If the FHR does not resolve, the primary health care provider should be notified immediately. Although it is always a good idea to have extra help during any unanticipated obstetric event, this is not the most important nursing measure at this time. If the FHR were to continue in an abnormal or nonreassuring pattern, a cesarean section may be warranted. This would require the insertion of a Foley catheter; however, the physician must make that determination. Oxytocin may put additional stress on the fetus. (D)

A nurse teaches a pregnant woman about the characteristics of true labor contractions. The nurse evaluates her understanding of the instructions when the woman states: A. "True labor contractions will subside when I walk around." B. "True labor contractions will cause discomfort over the top of my uterus." C. "True labor contractions will continue and get stronger even if I relax and take a shower." D. "True labor contractions will remain irregular but become stronger."

True labor contractions occur regularly, become stronger, last longer, and occur closer together. They may become intense during walking and continue despite comfort measures. Typically, true labor contractions are felt in the lower back, radiating to the lower portion of the abdomen. During false labor, contractions tend to be irregular and felt in the abdomen above the navel. Typically the contractions stop with walking or a change of position. (C)

A nurse teaches a pregnant woman about the characteristics of true labor contractions. The nurse verifies her understanding of the instructions when the woman makes what statement? A. "True labor contractions will subside when I walk around." B. "True labor contractions will cause discomfort over the top of my uterus." C. "True labor contractions will continue and get stronger even if I relax and take a shower." D. "True labor contractions will remain irregular but become stronger."

True labor contractions occur regularly, become stronger, last longer, and occur closer together. They may become intense during walking and continue despite comfort measures. Typically, true labor contractions are felt in the lower back, radiating to the lower portion of the abdomen. During false labor, contractions tend to be irregular and felt in the abdomen above the navel. Typically, the contractions often stop with walking or a change of position. (C)

The nurse educator is teaching a group of student nurses about the manifestations of true labor. Which statement by a student about true labor indicates effective learning? A. "The fetus is usually not engaged in the pelvis." B. "The cervix is often soft and is felt in the posterior position." C. "Contractions become more intense with walking." D. "Contractions are felt above the navel."

True labor is associated with painful contractions that become more intense as the client walks. These contractions indicate true labor. The fetus is engaged in the pelvis during true labor. When a client is in true labor, the cervix is becomes soft and is placed anteriorly. The fetus shows progressive change and is felt in the anterior position during true labor. For most women, the contractions of true labor are not felt above the navel but at the lower portion of the abdomen. (C)

*Challenge* not in book What should the nurse assess while reviewing the umbilical cord acid-base report of a client in order to determine placental function? A. Umbilical vein report B. Umbilical artery report C. Amniotic fluid concentration D. Blood glucose concentration

Umbilical acid base determination helps to determine the immediate condition of the newborn after birth. The nurse should check the umbilical vein report to determine the placental functioning of the client. The placental function would help to assess the nutritional status of the newborn baby. The umbilical artery report is used to determine the physiologic functioning of the newborn. Amniotic fluid concentrations are monitored during labor, but not after birth. Blood glucose levels in the fetus are not assessed with the umbilical cord test. (A)

When managing the care of a woman in the second stage of labor, the nurse uses various measures to enhance the progress of fetal descent. These measures include which actions? A. Encouraging the woman to try various upright positions, including squatting and standing B. Telling the woman to start pushing as soon as her cervix is fully dilated C. Continuing an epidural anesthetic so that pain is reduced and the woman can relax D. Coaching the woman to use sustained, 10- to 15-second, closed-glottis bearing-down efforts with each contraction

Upright positions and squatting may enhance the progress of fetal descent. Many factors dictate when a woman will begin pushing. Complete cervical dilation is necessary, but it is only one factor. If the fetal head is still in a higher pelvic station, the physician or midwife may allow the woman to "labor down" (allowing more time for fetal descent, thereby reducing the amount of pushing needed if she is able. The epidural may mask the sensations and muscle control needed for the woman to push effectively. Closed-glottic breathing may trigger the Valsalva maneuver, which increases intrathoracic and cardiovascular pressures, reducing cardiac output and inhibiting perfusion of the uterus and placenta. In addition, holding the breath for longer than 5 to 7 seconds diminishes the perfusion of oxygen across the placenta, resulting in fetal hypoxia. (A)

When managing the care of a woman in the second stage of labor, the nurse uses various measures to enhance the progress of fetal descent. These measures include: A. Encouraging the woman to try various upright positions, including squatting and standing. B. Telling the woman to start pushing as soon as her cervix is fully dilated. C. Continuing an epidural anesthetic so that pain is reduced and the woman can relax. D. Coaching the woman to use sustained, 10- to 15-second, closed-glottis bearing-down efforts with each contraction.

Upright positions and squatting may enhance the progress of fetal descent. Many factors dictate when a woman will begin pushing. Complete cervical dilation is necessary, but it is only one factor. If the fetal head is still in a higher pelvic station, the physician or midwife may allow the woman to "labor down" (allowing more time for fetal descent, thereby reducing the amount of pushing needed) if she is able. An epidural may mask the sensations and muscle control needed for the woman to push effectively. Closed-glottic breathing may trigger the Valsalva maneuver, which increases intrathoracic and cardiovascular pressures, reducing cardiac output and inhibiting perfusion of the uterus and placenta. In addition, holding the breath for longer than 5 to 7 seconds diminishes the perfusion of oxygen across the placenta, resulting in fetal hypoxia. (A)

Which of the following statements is not used to describe a characteristic of a uterine contraction? A. Frequency (how often contractions occur) B. Intensity (the strength of the contraction at its peak) C. Resting tone (the tension in the uterine muscle) D. Appearance (shape and height)

Uterine contractions are described in terms of frequency, intensity, duration, and resting tone. (D)

Which of the following statements is used to describe a characteristic of a uterine contraction? Select all that apply. A. Frequency (how often contractions occur) B. Intensity (the strength of the contraction at its peak) C. Resting tone (the tension in the uterine muscle between contractions) D. Appearance (shape and height) E. Attitude (the way the uterus presents itself)

Uterine contractions are described in terms of frequency, intensity, duration, and resting tone. Appearance is not a term used to describe contractions. Duration is another characteristic of uterine contractions. Uterine contractions are described in terms of frequency or how often the contractions occur. Uterine contractions are described in terms of intensity (the strength of the contraction at its peak). Uterine contractions are described in terms of resting tone (the tension in the uterine muscle) or attitude (the way the uterus presents itself). (A, B, C)

Fetal circulation can be affected by many factors during labor. Accurate assessment of the laboring woman and fetus requires knowledge of these expected adaptations. Which factor will affect fetal circulation during labor? Select all that apply. A. Fetal position B. Uterine contractions C. Blood pressure D. Umbilical cord blood flow E. Fetal sex

Uterine contractions during labor tend to decrease circulation and subsequent perfusion. Most healthy fetuses are well able to compensate for this stress and exposure to increased pressure while moving passively through the birth canal during labor. Maternal blood pressure is likely to have a significant effect on fetal circulation. Compression of the cord and reduction of umbilical blood flow affect fetal circulation. Maternal position may affect fetal circulation; however, fetal position is unlikely to disturb umbilical blood flow. The fetal sex does not affect umbilical blood flow. (B, C, D)

Under which circumstance would a nurse perform a vaginal examination on a client in labor? Select all that apply. A. On admission to the hospital at the start of labor B. When accelerations of the fetal heart rate (FHR) are noted C. On maternal perception of perineal pressure or the urge to bear down D. When membranes rupture E. During the duration stage of labor

Vaginal examinations should be performed when the woman is admitted to the hospital or birthing center at the start of labor. When the woman perceives perineal pressure or the urge to bear down is an appropriate time to perform a vaginal examination. After rupture of membranes (ROM) a vaginal examination should be performed. The nurse must be aware that there is an increased risk of prolapsed cord immediately after ROM. An accelerated FHR is a positive sign; variable decelerations, however, merit a vaginal examination. There isn't a duration stage of labor. (A, C, D)

Fetal monitoring of a pregnant client revealed that the fetal heart rate has minimal variability. Which prescribed drug is most likely responsible for the condition? A. Hydroxyzine (Vistaril) B. Terbutaline (Brethine) C. Secobarbital (Seconal) D. Atropine (Sal-Tropine)

Variability in the fetal heart rate can be classified as absent, mild, or moderate variability. This results in hypoxia and metabolic acidemia in the fetus. Central nervous system (CNS) depressants, such as secobarbital (Seconal), cause decreased variability in the fetal heart rate. This medication affects the baseline heart rate in the fetus by less than 5beats/minute. Hydroxyzine (Vistaril), terbutaline (Brethine), and atropine (Sal-Tropine) may result in tachycardia in the fetus. These drugs can increase the baseline fetal heart rate as much as 25 beats/minute. (C)

While monitoring the fetal heart rate, the nurse instructs the client to change her position and lie in the knee-to-chest position. What is the reason for the nurse to give this instruction to the client? A. Late decelerations in the fetal heart rate. B. Variable decelerations in the fetal heart rate. C. Early decelerations in the fetal heart rate. D. Prolonged decelerations in the fetal heart rate.

Variable decelerations in the fetal heart rate are usually caused by umbilical cord compression. The knee-to-chest position is useful for relieving cord compression, and thus the nurse should ask the client to move into this position. Prolonged decelerations in the fetal heart rate are not affected by the mother's position. If the nurse finds late decelerations in the fetal heart rate, the nurse should ask the mother to lie in the lateral position. Early decelerations in the fetal heart rate are a normal finding, and no nursing intervention is required. (B)

While assessing a pregnant client who is in labor, the nurse observes W-shaped waves on the fetal heart rate monitor. What would the nurse infer from this observation? A. Placental abruption B. Dilated cervical layers C. Umbilical cord compression D. Elevated uterine contractions

W-shaped waves in the FHR monitor are indicative of variable decelerations in the FHR. Variable decelerations are seen when the umbilical cord is compressed at the time of labor. Placental abruption and dilated cervical layers do not cause variable decelerations, but may cause late decelerations. Similarly, increased rate of UCs may also cause late decelerations in FHR. (C)

In which condition does the nurse document the fetal heart rate (FHR) tracing as category III? A. Prolonged FHR decelerations B. Presence of a sinusoidal pattern C. Recurrent variable decelerations D. Moderate baseline FHR variability

When categorizing the FHR tracing, the presence of a sinusoidal pattern meets the criteria for a category III, or abnormal, tracing. Immediate evaluation and prompt intervention is required when these patterns are identified. Prolonged FHR decelerations greater than or equal to 2 minutes but less than 10 minutes are category II FHR tracings. Recurrent variable decelerations accompanied by minimal or moderate baseline variability are also categorized as a category II FHR tracing. Category II FHR tracings are indeterminate and require continued observation and evaluation. An FHR tracing with moderate baseline FHR variability is categorized as category I, or normal. (B)

The nurse is briefing a client who is pregnant for the first time about "lightening." Which statement should the nurse mention to describe lightening to the client? A. Occurs when true labor is in progress. B. Allows the client to breathe more easily. C. Decreases the pressure on the bladder. D. Leads to decreased urinary frequency.

When the fetal head descends into the true pelvis during "lightening," the client will feel less congested and can breathe more easily. In a first-time pregnancy, lightening occurs about 2 weeks before term. In a multiparous pregnancy, lightening may not take place until after the uterine contractions are established and the true labor is in progress. This shift increases the pressure on the bladder and causes a return of urinary frequency. (B)

A client who is full term has experienced breathlessness throughout the pregnancy. The client reports a sudden ease in breathing, but also a frequent urge to urinate. What does the nurse interpret from these findings? A. The fetus has a vertex presentation. B. The client's cervix is dilated and effaced. C. The client has a rupture of fetal membranes D.The presenting part of fetus is engaged in the pelvis.

When the fetus descends down and engages in the pelvis, the diaphragm may get sufficient space to contract and relax, making it easier for the client to breathe. The lower position of the fetus in the uterus may put additional pressure on the bladder, however, increasing the frequency of urination. Based on the client's signs, it can be inferred that the fetus is engaged in the pelvis. However, the presenting part of the fetus cannot be determined by the client's signs. The presenting part of the fetus can be determined by palpation. When there is rupture of fetal membranes, there is dribbling of amniotic fluid. Cervical dilatation and effacement is indicated by a bloody cervical show. (D)

During the vaginal examination of a laboring client, the nurse analyzes that the fetus is in the right occiput anterior (ROA) position at -1 station. What is the position of the lowermost portion of the fetal presenting part? A. 2 cm above the ischial spine. B. 1 cm above the ischial spine. C. at the level of the ischial spine. D. 1 cm below the ischial spine.

When the lowermost portion of the presenting part is 1 cm above the ischial spine, it is noted as being minus (-)1. When positioned 2 cm above the ischial spine, it is -2 station. At the level of the spines the station is referred to as 0 (zero). When the presenting part is 1 cm below the spines, the station is said to be plus (+)1. (B)

While assessing a pregnant client using a fetoscope, the nurse also palpates the abdomen of the client. What is the purpose of palpating the abdomen of the client? A. Detection of fetal heart rate deceleration B. Evaluation of the severity of the pain caused by active labor C. Assessment of pain from pressure applied by the fetoscope D. Assessment of changes in fetal heart rate during and after contraction

While assessing the FHR with a fetoscope the nurse palpates the abdomen of the fetus to evaluate uterine contractions (UCs). This is done to detect any changes in the FHR during and after UCs. FHR decelerations are not identified by palpating the abdomen. They are assessed using the electronic fetal monitoring system. Pain perception is a subjective assessment. Moreover, the pressure due to the fetoscope is very minimal and does not cause pain. (D)


संबंधित स्टडी सेट्स

Unit 2 Module 5 (Scientific Method, Case Study, Naturalistic Observation, Surveys, & Random Sampling)

View Set

Unit three, Chapter 15, AP Euro Quiz

View Set

Ms studies Unit 4 focus questions

View Set

Nature and Environment 9 клас

View Set

Power Platform PL-900 Study Assessment

View Set